You are on page 1of 52

UIC PharmChem CARES A.Y.

2020-2021
(Academics Area)
GREEN REVIEW 2021

PHARMACOLOGY I
Note:
This reviewer is based on Katzung’s Basic and Clinical Pharmacology along with notes and handouts from discussions. The Green
Review 2021 reviewers were authored by chosen pharmacy students of the University of the Immaculate Conception (UIC) and were
checked by the faculty of UIC’s College of Pharmacy and Chemistry (CPC) and, however, may still be subject to minor errors. The Green
Review 2021 reviewers are distributed under the name of CPC’s academic club, the PharmChem CARES, under no circumstance, are in
collusion with UIC’s CPC administration concerning the contents of the upcoming comprehensive exams. Thank you.
UIC PharmChem CARES
GREEN REVIEW 2021: Pharmacology I 2

PHARMACOLOGY → Cause: intolerance, idiosyncrasy, allergy, genetic


→ Reason/ further cause: overdose, accident, failure of
→ Study of substance that interact with living systems expected action, withdrawal, events not included in
through chemical process by binding to regulatory labelling
molecules and activating or inhibiting normal body
processes MAJOR EXPANSION IN HISTORY OF PCOL
→ Study of DRUGS – agents used in prevention,
diagnosis, mitigation, cure, treatment 1. Drug action and receptors, molecular basis
2. New drugs and drug groups
PRINCIPLES OF PHARMACOLOGY 3. MOAs
4. Isolation of receptors
1. All substances can be toxic - Orphan receptors
- Chemicals in botanicals (herbs, plant extracts,
nutraceuticals) are not different from chemicals
in manufacturing drugs BRANCHES OF PCOL
2. All dietary supplements and all therapies promoted as
health enhancing should meet the same standards of 1. Toxicology
efficacy and safety as conventional drugs and → Undesirable effects of chemicals on living
medical therapies systems from individuals cells to humans to
complex ecosystems
HISTORY → Ex. Cadmium (Cd) = itai itai disease
2. Pharmacogenomics/ pharmacogenetics
De Materia Medica → Genetic variation that causes differences in
→ By DIOSCORIDES drug response among individual and
→ Early written records population
→ No further understanding of MOA 3. Pharmacotherapeutics
→ No methods of purifying APIs → Use of drugs to prevent/ treat disease
→ Lack of hypotheses about drug’s actions
→ Science of drug preparation and medical uses of → “stranger” “xeno ba sila?”
drugs → Chemicals not synthesized
→ PCOL precursor by body
▪ Drugs
* Pharmaceutical
Late 18th And 19th Centuries industry
XENOBIOTICS
* Hospitals
→ FRANCOIS MAGENDIE and CLAUDE BERNARD
▪ Pesticides
- Developed methods of experimental physiology
and pharmacology * Fungicides
* Agri
▪ Chemicals
unscientific worthless controlled * Chemical industry
claims "patient" clinical trials
(anonymous) medicines (4) HORMONES → Drugs synthesized by body
→ Almost exclusively have
harmful fx in the body
→ PARACELSUS:
POISON
PHASES OF CLINICAL TRIALS “the dose makes the
Phase 1 • 20 -100 subjects poison”
• Evaluate safety of medicines ▪ Inorganic like Pb, Hg
• Determine safe dosage → Biological
• Identify side effects TOXINS → Synthesized by plants and
Phase 2 • 100- 200 subjects animals
• Further evaluate safety
• Test effectiveness
GENERAL PRINCIPLES OF PCOL
Phase 3 • 1000- 6000 subjects
• Confirm effectiveness Agonists
• Monitor side fx → Drugs that occupy receptors and activate
• Fever and muscle pain
• Compare to other treatment Antagonists
• Collect information → Drugs that occupy receptors but do not activate them
Phase 4 • Post marketing surveillance → Block receptors activation of agonists
Ex. Paracetamol,
> 4g/ day = risk of liver
tox.
• Provide additional info after
approval (risk, benefits, use)
ADVERSE DRUG REACTION (ADR)

→ 4th leading cause of death


Prepared by: MJCN
UIC PharmChem CARES
GREEN REVIEW 2021: Pharmacology I 3

c. D + R → DR complex→ coupling molecule activation


→ effector molecule = fx
(-) of metabolism of endogenous activator → 
activator action on effector molecule =  fx

VII. DRUG REACTIVITY & RECEPTOR BONDS

3 MAJOR TYPES:

1. Covalent
→ Strongest force
→ Acetylsalicylic acid (ASA) or aspirin
I. PHYSICAL NATURE OF DRUGS - Antiplatelet/ anti-coagulant
→ Solid, liquid, gas 2. Electrostatic
→ Determines ROA (Route of administration) → Van Der Waals Forces
- Administered distant to its site of action → more common than covalent in drug- receptor
→ Weak acids/ bases interaction
→ weaker than covalent
3. Hydrophobic
→ Quite weak
II. DRUG SIZE → Important in highly lipid- soluble/ lipophilic drugs
→ Very small to very large MW
→ Mostly MW 100- 1000 SIGNAL TRANSDUCTION PATHWAY
• Lithium ion
- Bipolar disorder 1. Signalling molecule
- Very small drug → Drug
- MW 7 → Ligand- any substance that binds to receptor
• Alteplase → @ ECF
- Thrombolytic/ prevents clot formation
- Very large drug 2. Receptor
- MW 59050 → Signalling detector
→ @ Plasma membrane
III. DRUG SHAPE 3. Molecules
→ Permits binding to receptor → Signal transduction
1. CHIRALITY (STEREOISOMERISM) → Messengers
▪ Carvedilol → @cytoplasm
- ACE inhibitor 4. Cellular response
- (S-) more potent than → Effect
- (R+) enantiomer → @cytoplasm
▪ Ketamine
- IV anesthetic Two Important Features
- (R+) is more potent than (S-) enantiomer 1. Ability to amplify signals
2. STEREOSELECTIVITY OF ENZYMES 2. Mechanism to protect cells from excessive
stimulation
IV. RATIONAL DRUG DESIGN
→ Appropriate molecular structure basis of its
receptor
→ Computer program

V. RECEPTOR NOMENCLATURE
→ (IUPHAR) International Union of
Pharmacology committee on receptor
nomenclature

VI. DRUG- BODY INTERACTION


◆ Pharmacokinetics
→ LADME/ ADME
◆ Pharmacodynamics
→ Drug action and mechanism
→ Drugs + receptor or cellular component = response/
drug fx
a. D + receptor- effector → DRE complex = fx
b. D + R → DR complex = fx
Prepared by: MJCN
UIC PharmChem CARES
GREEN REVIEW 2021: Pharmacology I 4

(2) Beta- blocker


(3) Potassium
(4) Calcium
ii. Carriers
iii. Enzymes
iv. Receptors
(1) Ligand-gated ion channel
(2) G protein coupled receptor
(3) Enzyme linked
(4) Intracellular receptor

1. Enzymes

• Cells have many different types of


receptor, each is specific for a
particular agonist and produces a
unique response.
• Magnitude of response depends on
the number of drug-receptor complex.

→ Solution: give HMG- CoA reductase inhibitor like


DRUG RECEPTORS
STATINS- inhibit cholesterol synthesis
2. Nucleic Acids
Receptors → DNA and RNA
✓ Largely determine the quantitative relations between
dose and pharmacologic fx (CONCENTRATIONS) 3. Proteins
✓ responsible for selectivity of drug action (AFFINITY)
✓ Mediate the actions of pharmacologic actions of a. Structural Proteins
agonists and antagonists
→ Based on cell structure:
Major Receptor Families
1. Enzymes 1. MICROTUBULES
2. Nucleic Acids - Cell division and mitosis
3. Proteins - Inhibited by ANTIMITOTICS or
a. Structural CHEMOTHERAPETIC AGENTS:
b. Regulatory ▪ Taxanes- paclitaxel
i. Channels (“so be pota ca?”) ▪ Vincas- Vincristine
(1) Sodium - MOA: affect actively dividing cell

Prepared by: MJCN


UIC PharmChem CARES
GREEN REVIEW 2021: Pharmacology I 5

- antidepressants
Inhibit mitotic spindle ▪ Dopamine
 - Psychostimulant
No cell division - cocaine

No production of malignant cells 3. ENZYMES
2. TUBULIN
→ Anti-inflammatory ▪ CYCLOOXYGENASE (COX)
→ Colchicine - Inflame, pain
- Inhibited by NSAIDs – anti-inflammatory agents
(ibuprofen, mefenamic acid)
b. Regulatory Proteins
→ Neurotransmitters, hormones, autacoids
4. RECEPTORS
1. CHANNELS
→ Associated with ions channels
Na+, K+, Cl-
A. DQP (“duck, quack, pato”) → Hydrophilic and hydrophobic
ligands
▪ Disopyramide
▪ Quinidine ▪ GABA (Gamma Amino Butyric
▪ Procainamide Acid)
- Major in brain
- Drugs that act on GABA
receptor:
B. TMLP (“too much love is
o BenzodiaZepines
VOLTAGE GATED NA+ painful)
-  frequenZ
CHANNEL
o BarbituRATes
▪ Tocainide
Inhibit Class 1 Anti- ▪ Mexiletine -  duration
Arrythmics Anxiety and Panic disorder
▪ Lidocaine
▪ Phenytoin →  excitation of CNS
LIGAND –GATED ION
CHANNELS → Moa of barbiturates:
C. MFP (“More Fries Pls”) * Barbiturates bind to GABA
Aka IONOTROPIC receptor
▪ Moricizine * Influx of Cl- ions
▪ Flecainide * HYPERPOLARIZATION or
▪ Propafenone  - ions
* Relaxation
* (“the irony of more nega
▪ Dronedarone more relaxed”)
VOLTAGE GATED K+
▪ Amiodarone ▪ GLYCINE
CHANNELS ▪ Dofetilide
- Major in spinal cord
Inhibits Class 3 Anti- ▪ Bretylium
▪ 5- HT3 (5-
Arrythmics
▪ Ibutilide
▪ Sotalol HYDROXYTRYPTAMINE TYPE
3)
- Inhibits “SETRON”
Ondansetron
VOLTAGE GATED (Ver Di) chemotherapeutic
Ca2+ CHANNELS
▪ Verapamil o  concentration of metabolites
Inhibits Class 4 Anti- ▪ Diltiazem - 2° messengers
Arrythmics
G- PROTEIN
COUPLED TYPES OF G PROTEINS
2. CARRIERS
RECEPTOR o Gs
- Stimulates adenyl
Na+/ K+ ATPase pump Aka Metabotropic,
cyclase
→ Na+ in; K+ out Seven
transmembrane, AC
INHIBITORS OF Na+/ K+ ATPase pump: Serpentine ATP Camp
▪ Digoxin (Digitalis lanata)
▪ Digitoxin (Digitalis purpurea) o Gi
- Toxic - Inhibit adenylyl cyclase
▪ Norepinephrine and Serotonin
Prepared by: MJCN
UIC PharmChem CARES
GREEN REVIEW 2021: Pharmacology I 6

AC SUMMARY
ATP cAMP
TYPE 1
Ligand- gated ion Cell membrane Milliseconds
channel
o Gq
- Stimulates TYPE 2
phospholipase C G- protein coupled Cell membrane Seconds
receptor
Phosphatidyl inositol- 4,5- TYPE 3
biphosphate Cell membrane Minutes
Enzyme-linked
TYPE 4
PLC Nucleus/
Intracellular hrs
cytoplasm
IP3 (inositol-1,4,5- triphosphate) Receptor

DAG (Diacyl glycerol)

FEATURES OF LIGANDS
Effects:

*  intracellular Ca2+ Affinity


phosphorylation of myosin → Ability to bind to receptor
light chains = SMOOTH
MUSCLE CONTRACTIONS Intrinsic Activity
(heart) → Constitutive/ endogenous activity of receptor
* Problems in Gq = heart failure → Effect of a receptor even in the absence of ligand

Gq α 1 urinary retention Allosteric Site


i α 2 relaxation for → Site other than ligand binding site/ active site
anxiety ▪ Alloteric activator/ allosteric agonist
s β 1  HR - Enhances ligand-binding
s β2 ▪ Allosteric Inhibitor/ Allosteric antagonist
Gq M 1 urinary retention - Inhibit ligand binding
i M2
q M 3 urinary retention
Gs D1
i D2
q H1 V1
s H2 V2

Secondary messengers:

o cAMP (Cyclic adenosine


monophosphate)
o cGMP (cyclic guanosine
monophphspate)
o IP3 (Inositol-1,4,5-
triphosphate)
o DAG (Diacyl glycerol)

Prepared by: MJCN


UIC PharmChem CARES
GREEN REVIEW 2021: Pharmacology I 7

CLASSIFICATIONS OF LIGANDS • Slope: Change in response with change in dose

AGONIST

→ Produces all effect of receptor


→ Morphine
FULL
- Binds to MU receptor
- Analgesic with sedative fx

→ Produces some fx of a receptor


→ Cloniphine citrate
PARTIAL - Fertility drug
- Partial agonist of estrogen
receptor

INVERSE → Decreases intrinsic activity 2. QUANTAL DOSE


ANTAGONIST → Plot: cumulative # px VS log dose
→ Can be fully overcome → Parameters:
by increasing agonist
COMPETITIVE
concentration ▪ ED50 (Median Effective Dose)
→ reversible - Dose that produces beneficial responses in 50%
→ cannot be fully population
NON- COMPETITIVE overcome by  agonist ▪ TD50 (Median Toxic Dose)
concentration - Dose that produces toxic responses in 50%
→ two ligands acting on population
different target ▪ Therapeutic index
producing opposite - Measures relative safety
effect TD50 LD50
- = ED50
ED50
ex. GLUCOCORTICOIDS ▪ Margin of safety
- NCR TYPE 3 - How safely can we take drug for therapeutic
FUNCTIONAL/
-  blood sugar purpose without resisting adverse effects
PHYSIOLOGICAL
ANATGONISM
INSULIN
- β2 pancreas
-  blood sugar

Insulin must be taken with


glucocorticoids

THEORIES

o HYPOTHESIS OF CLARK
→ Fx are produced when all receptors are
occupied
o HYPOTHESIS OF PATON
→ Rate theory – stimulus- response
o HYPOTHESIS OF ARIENS AND STEPHENSON
→ Occupancy theory
→ Effects last as long as receptors are
occupied 3. DURATION OF ACTION
→ Drug takes effect only when the drug occupies the
receptor
RELATIONSHIP BETWEEN DRUG CONC. AND RESPONSE → Some cases wherein effects still do persist
because of coupling mechanism is still present for
1. GRADED DOSE the activated form
→ Plot: response VS log dose
→ Parameters: 4. RECEPTOR AND INERT BINDING SITES
• Efficacy: Maximum achievable response ▪ Selectivity
• Ceiling dose: Smallest dose that gives effect ▪ Can change its function
• Potency: Dose needed to achieve 50% response → Inert Binding Sites: endogenous molecule that are
capable of binding drugs other than the receptors.
o Albumin
o Alpha-1-Acid GP
Prepared by: MJCN
UIC PharmChem CARES
GREEN REVIEW 2021: Pharmacology I 8

iii. Release of histamine


5. DESENSITIZATION & DOWN-REGULATION OF
RECEPTORS TRIPLE RESPONSE OF LEWIS:
→ Repeated & Continuous Administration of drug → Intradermal injection of histamine causes:
→ To prevent potential damage to the cell ▪ Red spot
a. Desensitized ▪ Edema
- Diminished effect/ rapidly diminishing effect or
▪ Flare response
response due to successive dose of drugs
- Tachyphylaxis
4 TYPES OF RECEPTORS
b. Down-Regulation:
- Receptors are unavailable for interaction
- Prolonged exposure to  concentration of 1. HISTAMINE 1 (H1)
agonist causes a reduction in no. of receptors
available for interaction Location: Effect:
- Endocytosis of receptors from cell surface Vascular smooth muscle Vasodilation
o Recycled - restored activity  HR
o Degraded - total number of receptors Bronchial smooth muscle Broncho-constriction
Sensory nerve endings Pain, itchiness,
inflammation
CNS Wakefulness
AUTACOIDS Endothelial lining Contraction
→ Greek: autos = SELF; acos = RELIEF/ DRUG Drugs: H1 RECEPTOR ANTAGONIST/ ANTIHISTAMINES
→ Local hormones- like 1ST Generation 2ND Generation
→ Biological factors - Old drugs - less sedating
→ Site of release is near the site of action/ synthesis - Highly sedating - true non-
→ Brief duration - Crosses BBB sedating
→ Not blood borne - Lipophilic drugs
→ Histamine, serotonin LESS SEDATING
ETHANOLAMINE • cetirizine
(Most sedating) • acrivastine
HISTAMINE
• Diphenhydramine
• Dimenhydrinate TRUE NON- SEDATING
→ Important mediator of immediate allergic and • Doxylamine • Loratadine
inflammation reaction • Carbinoxamine (Claritin ®)
→ occurs in plant and animal tissues, venoms and Desloratadine
stings PIPERAZINE (Aerius ®)
→ Gastric acid secretion (anti-motion sickness) • Fexofenadine
→ Formed by decarboxylation of L- histidine • Meclizine (Bonamine
®)
L- Histidine
• Cyclizine
• Hydroxyzine
L- histidine decarboxylase Decarboxylation
ETHYLENEDIAMINE
• Pyrilamine
HISTAMINE • Tripelenamine

WAYS THAT TRIGGER HISTAMINE RELEASE ALKYLAMINE


(cold tx.)
1. Allergic Reaction • Brompheniramine
i. Antigens bind to B cells • chlorpheniramine
ii. B cell activates plasma cell
iii. plasma cell release IgE PHENOTHIAZINE
iv. IgE binds to mast cells (antipsychotics)
v. Mast cells release histamine • promethazine

PIPERIDINE
2. Tissue injury
• cyproheptadine
i. Introduction of pathogen in the mast cell found in
- tx. for serotonin
skin syndrome (changes in
ii. mast cells release histamine mental state,
hypothermia)
3. Drugs and foreign chemicals * DOC [serotonin
i. Drugs/ foreign chemicals (venoms, dyes, syndrome]: anti-
alkaloids, morphine) serotonergic (s/e is
ii. Displacement of histamine in mast cell
Prepared by: MJCN
UIC PharmChem CARES
GREEN REVIEW 2021: Pharmacology I 9

malignant → When blood is allowed to clot, serotonin is released


hyperthermia) from clot to serum
* DOC [malignant → A neurotransmitter, a local hormone in the gut,
hyperthermia] component of platelet clotting process
dantrolene → Play a role in migraine and carcinoid syndrome

TRYPTOPHAN
HISTAMINE 2 (H2) Tryptophan
2.  Hydroxylation
Inverse agonist hydroxylase
Location: Effect:
Parietal cell of stomach Basal gastric acid 5-HYDROXYTRYPTOPHAN
secretion
Drugs: H2 RECEPTOR ANTAGONIST/ “TIDINEs”/ H2  Decarboxylation
BLOCKERS
Action: gastric acid secretion SEROTONIN/ 5HT3
Advantage: toxicity
• Famotidine (H2 Bloc®) PHYSIOLOGIC DISTRIBUTION
- Most potent; PO
• Cimetidine (Tagamet ®) CNS
- Least potent; PO 10%
Neurons
- Anti- adrogenic: S/E is GYNECOMASTIA (breast GI Tract
enlargement among male ) 90%
Enterochromaffin- like cells
* DOC: ketoconazole, spironolactone
- s/e: menstrual irregularity
BIOSYNTHESIS
• Nizatidine (Axid ®)
- Most bioavailable; IV
- Ranitidine (Zontac ®)- PO SITE PERIPHERAL EFFECTS
Brain Vasoconstriction/ relaxation
HISTAMINE 3 (H3) Small intestine GI peristalsis
3.
(no available general use) Platelets Platelet aggregation
- Homologous w/ H4 receptors and have some affinity GENERAL EFFECTS
to it CNS mood regulation, BP
- For sleep disorders Temperature regulation,
- Narcolepsy emesis
- Obesity
- Cognitive and psychiatric disorders
RECEPTORS
Tripolisant
- Inverse H3 agonist
- Reduce sleep cycles in humans w/ narcolepsy
Pitolisant 1. 5HT 1A
- Reduce drowsiness in narcolepsy px LOCATION EFFECTS
Presynaptic receptor (brain)
Hippocampus Relaxation of CNS
Raphe nuclei
HISTAMINE 4 (H4)
4. Drugs: 5HT 1A AGONIST
(no available general use)
- H1 blockers have more affinity on H4
Buspirone
H4 BLOCKERS: - Tx. for anxiety ( condition characterized by  CNS
- Potential in chronic inflammatory conditions like activity)
asthma
- Useful in pruritus, asthma, allergic rhinitis, and pain
conditions
2. 5HT 1B/ 1D
LOCATION EFFECTS
SEROTONIN Peripheral blood vessels
Substantia nigra
→ Aka 5-HYDROXYTRYPTAMINE, 5HT3 Brain Vasoconstriction
→ 5- HYDROXYTRYPTOPHAN metabolites: Globus pallidus
Hydroxy = enteramine Basal ganglia
Tryptophan = serotonin Drugs: 5HT 1B/ 1D AGONIST/ “TRIPTANS”
→ Powerful vasoconstrictor except in skeletal and - Intranasal/ sublingual
cardiac muscles
Prepared by: MJCN
UIC PharmChem CARES
GREEN REVIEW 2021: Pharmacology I 10

Managaement of Acute
• Sumatriptan Migraine HA EICOSAINOIDS
• Zolmitriptan - Too much pressure on
→ eicosa tetraenoic acid
• naratriptan blood vessel
→ Eicosa= 20 C
- Vasodilated
→ Tetra = 4 (5,8,11,4)
→ En= alkene
1. Prostaglandin
3. 5HT 2A 2. Thromboxanes
3. Leukotrienes
LOCATION EFFECTS
Vascular smooth muscle vasoconstriction
Uterus contraction PATHOPHYSIOLOGY
Platelets
Cerebral cortex During tissue injury,

Drugs: 5HT 2A AGONIST


Phospholipids
secretes
ErgotaMine
- tx for Migraine HA
- Agonist; vasoconstriction
Phospholipase
ErgonoVine a2
- Mgmt.. of post partum bleeding (dilation) (Vagina)
- Agonist; contraction
arachidonic
Methysergide
- Prophylaxis in migraine acid
- Antagonist; dilation
- Withdrawn due to s/e:
• Retroperitoneal fibrosis
-  fibrous tissue → scarring LOX COX
- idiopathic
• Retropulmonary fibrosis
- idiopathic
LOX=
COX 1 COX 2

4. 5HT 3 Lipoxygenase
LOCATION EFFECTS COX = Cycloxygenase

Area postrema (CNS) COX 1


activates (radiation) Emesis/ vomiting → Plrimarily responsible for production of
chemoreceptor trigger zone PROSTAGLANDIN & THROMBOXANES
GIT • Secrete gastric
Eneteric and sensory nerves mucus
• Regulate gastric acid
Drugs: 5HT 3 ANTAGONIST/ CHEMOTHERAPEUTIC LOCATIONS to normal HCl level
AGENTS/ “SETRON”
Platelets Promotes aggregation
Palonosetron Kidneys Maintenance of renal
Granisetron blood flow
Inhibits emesis
Ondansetron COX 2
Dalansetron → Not expressed constantly in most patients
→ Induced by inflammation
5HT 4
→ Products:
LOCATION EFFECT • Prostaglandins (PGs):
GIT  motility - Mediate mainly inflammation and fever
Drug: 5HT 4 AGONIST

Tegaserod 1. NSAIDs
- tx. IBS (condition with constipated fx) → MOA: inhibit COX
→ Effects:

Prepared by: MJCN


UIC PharmChem CARES
GREEN REVIEW 2021: Pharmacology I 11

o  PG, TXA
Normally, PGI2 and TXA2 must be balanced
o Analgesic
▪ PGI2 effects:
o Antipyretic
o Vasodilation
o Anti-inflammatory
o Antiplatelet
Heart o Anti- aggregant
▪ Ketorolac ▪ TXA2 effects:
SELECTIVE COX-1
▪ Ketoprofen o Vasoconstriction
INHIBITORS
▪ Flurbiprofen o Proaggregant
- Bind to only 1 receptor
▪ Low dose ASA o Stroke, MI
▪ Meloxicam
SELECTIVE COX-2 ▪ Diclofenac 2. PROSTAGLANDIN ANALOGS
INHIBITORS ▪ Celecoxib
→ Mimic the endogenous prostaglandin
▪ Etodolac
→ Needs GPCR to exert fx
▪ Ibuprofen PGE 1 ANALOGS
▪ Diflunisal ▪ Alprostadil
▪ Piroxicam → Tx for erectile dysfunction
▪ Naproxen → Vasodilation fx
- Tx for malignant Moa:
fever Stimulate cAMP
NON- SELECTIVE COX - Test for cancer 
INHIBITORS - If px’s fever does intracellular Ca2+ levels
- Drugs can bind to more not get better 
than 1 receptors with ASA and Dilation of cavernosum arteries
paracetamol but 
responds to erection
naproxen then
fever is caused ▪ Lubiprostone
by cancer → Chronic constipation tx.
Moa:
SIDE EFFECTS OF NSAIDS
 gastric mucus (no protection in mucus lining) Activate type 3 chloride channels (epithelial linings in the
GIT intestine)
→  HCl secretion →  risk of GI bleeding and
peptic ulcer 
Secretion of Cl- ions

Proaggregant TXA2 + NSAIDs →  Passive secretion of Na+ and H2O
antiplatelet activity →  bleeding 
liquidity of intestinal content due to secretion of H2O
* Bleeding is evident to px. Taking 
aspirin Stimulate intestinal smooth muscle contraction
* Moa of aspirin is irreversibly 
Platelet inhibit COX in platelets  passage of stool
* Platelets do not have nucleus and
cannot make new enzymes ▪ Misoprostol (cytotec ®)
* Antiplatelet persists even when MOA 1: Prevent NSAID- induced peptic ulcer
the px stops aspirin therapy
* Platelets will take several days to Binds to prostaglandin receptor @ gastric parietal cell
replace the old ones 
 cAMP activity

Prostaglandin E2, Prostaglandin I2  proton pump activity
 
Vasodilation Inhibited
Inhibit acid secretion
 by
PGE2, PGI2 NSAIDs
MOA 2: Abortifacient; induce labor

Vasoconstriction PG receptor in uterus
Kidneys
 
Kidney injury Softening of cervix

▪ Vasodilation is important in
maintaining Glomerular filtration rate; Uterine contraction
once not maintained, backflow of 
wastes+ toxicities Easy expulsion of uterine contents

Prepared by: MJCN


UIC PharmChem CARES
GREEN REVIEW 2021: Pharmacology I 12

MOA 3. Protects stomach lining by  bicarbonates and ▪ Mefenamic acid


mucus production - Not take > 5 days
▪ Meclofenamide
- 8 hr interval
PGE 2 ANALOGS
- C/I: 14 yrs old
▪ Dinoprostone
children
→ Abortifacient (2nd trimester)
MOA: directly affects collagenase of cervix → softening of ▪ Naproxen
cervix → uterine contraction → expulsion ▪ Ibuprofen
Propionic acid derivatives
▪ Flubiprofen
PGE 2 α analogs ▪ ketoprofen
▪ Latanoprost
▪ Binatoprost
▪ Travopost LOX PATHWAY

MOA: IOP,  Aqueous outflow in the eye


USE: tx. for glaucoma; Via ophthalmically
PGI2 (PROSTACYCLIN) ANALOGS
▪ Iloprost activated
▪ Trepostonil arachidonic acid

MOA:
 Camp lipoxygenas
 e
 intracellular Ca2+ in pulmonary vascular smooth muscle
cell
 5-
Vasodilation Hydroxyperoxyeicosatetraeno
 ic acid (5-HPETE)
 pulmonary vascular resistance

cardiac index/ heart’s performance
LTA 4
USE: tx for pulmonary arterial hypertension

LTC 4 LTD 4 LTE 4 Lipoxins


NSAIDs CLASSIFICATIONS
General effects:
▪ Phenylbutazone
Pyrazolone derivatives ✓ smooth muscle contraction
▪ Oxyphenlbutazone
“butazone” ✓ bronchoconstriction
▪ sulfinpyrazone

ASTHMA
▪ indomethacin
→ airways are narrowed and eventually swell
- disadvantage:  risk of
Indole derivatives → produce mucus
NSAID-induced gastritis
- affinity: COX1 > COX2 → symptoms: wheezing
→ drugs:
Pyrole alkanoic acid • RELIEVERS: treat acute exacerbation of
▪ tolmetin asthma
derivatives
• CONTROLLERS: prevent/ control acute
exacerbation of asthma
▪ Piroxicam
- Non-selective COX • LEUKOTRIENE MODIFIERS:
Oxicam derivatives inhibitor 1. LOX inhibitor
-  risk for PUD/ gastritis ▪ Zileuton
- Affinity: COX1 > COX 2
2. LTD 4 receptor antagonist
(controllers)
▪ Diclofenac
Phenylacetic acid ▪ Montelukast
▪ Sulindac
derivatives ▪ Zafirlukast
▪ Nabumetone
Rare s/e:
• Primarily used as
✓ Churg strauss syndrome
analgesics
Fenamates (blood vessels inflammation)
• 1st or primary line for
management of pain ✓ Hepatitis
Prepared by: MJCN
UIC PharmChem CARES
GREEN REVIEW 2021: Pharmacology I 13

→ MAO (monoamine oxidase)


→ COMT (catechol-O-methyltransferase)
AUTONOMIC NERVOUS SYSTEM → Release of NTs occur

→ Effector of CNS: BRAIN and SPINAL CORD POST SYNAPSE


→ Regulate several involuntary actions → Most receptors are found
→ Metabolizing enzymes
central nervous peripheral
system nervous system DIVISIONS OF AUTONOMIC NERVOUS SYSTEM

Enteric Nervous system


sensory motor → Large collections of neurons
→ LOCATION: walls of digestive tract
→ Motility and secretions of GIT
→ 2 plexuses (networks of nerves)
somatic autonomic
o AUERBACH- myenteric
o MEISSNER- submucosal
→ Neurotransmitters:
sympathetic o VIP (vasoactive intestinal peptide)
division
(adrenergic) o ATP (adenosine triphosphate)
o Neuropeptide Y
o Substance P
parasympathetic
division SYMPA. PARASYMPA.
(cholinergic)
Anatomy
Origin Thoracolumbar Craniosacral
T1-T12, L1-L5 CN 3,7,9, 10, S2-S4
SOMATIC AUTONOMIC Length of fiber
# of neurons 1 2 Preganglionic Shorter Longer
Ganglion NO YES Postganglionic Longer shorter
Synapse 1 2 Location of Near spinal Near target organ
Activity VOLUNTARY INVOLUNTARY ganglia cord
Target organs Sweat glands, Cardiac and smooth
Synapses
* Somatic: cardiac and muscle, gland cells,
SOMATIC Skeletal smooth nerve terminals
Presynapse Somatic muscle muscle,
Postsynapse Neuromuscular end plate Gland cells,
nerve
ANS 1
terminals, renal
Presynapse Preganglionic fiber vascular
Postsynapse ganglion smooth muscle
ANS 2 Neurotransmitter
Presynapse Post ganglionic fiber Preganglionic Acetylcholine Acetylcholine
Postsynapse Target organ postganglionic NE, Epi, Acetylcholine
Dopamine
SYNAPTIC NEUROTRANSMISSION Receptors
→ Communication point between neuron-neuron or
Ganglia NN NN
neuron –target organ
Target organ α, β, D NM
→ Conduction of impulses
Responses Fight/ flight Rest and digest
eyes Dilation/ Constriction/ miosis
PARTS OF SYNAPSE mydriasis
1. Pre synapse Bronchi Bronchodilation Bronchoconstriction
2. Synaptic cleft Heart Tachycardia Bradycardia
3. Post synapse GIT wall Relaxation Contraction
GIT Sphincter Contraction/ Relaxation/open=
PRESYNAPSE close= diarrhea
→ Storage, synthesis, release of NTs constipation
→ Presynaptic receptors Urinary bladder
o NTs bind to them for autoregulation (Inhibit Detrusor Relax Contract
NT release) Trigone/ Contract/ Relax/ open=
SYNAPTIC CLEFT sphincter close= urinary urination/ micturition
→ Metabolizing enzymes are found retention

Prepared by: MJCN


UIC PharmChem CARES
GREEN REVIEW 2021: Pharmacology I 14

Sweat glands Apocrine (palm Eccrine (all over the Dopamine


and soles) body)
 VMAT
BIOSYNTHESIS OF CATECHOLAMINES Vesicle

Location: 
✓ Sympathetic post ganglionic fiber
NE
✓ CNS
✓ Adrenal medulla 
Precursor
✓ Tyrosine NE release

1. Active uptake of TYROSINE into presynaptic receptor NE & EPI


Channel: carrier- mediated Na+ channel
2. Tyrosine → DOPA (dihydroxyphenylalanine) 3. Bretylium, Guanethidine, Guanadrel
Enzyme: tyrosine hydroxylase → Inhibits exocytosis of catecholamines
3. DOPA → dopamine → Inhibit exocytotic release of NE
DOPA Decarboxylase → Bretylium: Class 3 Anti- Arrythmics
4. DOPAMINE enters vesicle → Guanethidine, Guanadrel: Anti- HTN
VMAT (vesicular monoamine transporter)
5. DOPAMINE → NOREPINEPHRINE
Dopamine B- hydroxylase
6. Exocytotic release of NE 4. Tyramine, Ephedrine, Amphetamine,
Channel: carrier mediated Ca2+ channel Methamphetamine (TEAM)
7. In adrenal medulla, NE → Epi → Stimulates exocytosis of catecholamines
Phenylethanolamine-N-methyltransferase Amphetamine, Methamphetamine
* Recreational drugs
* CNS stimulant for the tx of narcolepsy
FATES OF CATECHOLAMINES (excessive sleeping)

5. Cocaine, TCA, Reboxetine


✓ Binding to post-synaptic receptors
→ Inhibits reuptake of catecholamines
✓ Metabolism by MAO and COMT at post synaptic cleft
→ Cocaine
and synaptic cleft
o Recreational drug
✓ Reuptake into presynapse
o Intense feeling of happiness
Transporter: UPTAKE 1 TRANSPORTER/
o Loss of contact w/ reality
NE TRANSPORTER (70%)
o Strong CNS stimulant
→ Tricyclic antidepressant (TCA):
DRUGS
o Mgt of depression
→ Reboxetine
1. Metyrosine
o Anti- histamine
→ Inhibits Tyrosine Hydroxylase
→ DOPA = NE & EPI
→ used in treating PHEOCHROMOCYTOMA
ADRENERGIC RECEPTORS AND THEIR PHYSIOLOGICAL
(rare tumor in adrenal gland caused by excessive
EFFECTS
release of NE and EPI)
NE & EPI → Hormones that control HR, metabolism SYMPATHETIC = ADRENERGIC
and BP
* Most common symptoms: headaches, palpitations,
sweating

2. Reserpine
→ inhibits Vesicular Uptake of Catecholamines
→ from Rauwolfia serpentine
→ antihypertensive and antipsychotic
→ not available in USA because of serious s/e:

• depressions
• heart beat
• Nightmares
• Sympa fx.

Prepared by: MJCN


UIC PharmChem CARES
GREEN REVIEW 2021: Pharmacology I 15

EYES
Iris radial muscle α1 Pupillary DILATION RECEPTORS ORGANS EFFECTS
Ciliary muscle β2 RELAXATION for Iris radial Pupillary DILATION
FAR vision muscle
HEART Trigone CONTRACTION
AV Nodes β1, β2 DROMOTROPISM sphincter
• conduction Salivary  Secretions
activity glands
SA, AV Nodes β1, β2 Spontaneous Bronchial  secretions
depolarization α1 glands
Atria, ventricles β1, β2 INOTROPISM intestines motility
•  Strength of Arteries, CONSTRICTION
contraction arterioles via α1, β2
CHROMOTROPISM DILATION
Veins,
• Rate of via β2
venules
contraction
BLOOD VESSELS Stomach Motility secretions
Arteries, α1, α2, β2 CONSTRICTION Arteries, CONSTRICTION
arterioles via α1, β2 arterioles via α1, β2
Veins, venules α1, α2, β2 DILATION DILATION
via β2 α2 Veins, via β2
venules
URINARY BLADDER
Detrusor β2 RELAXATION Stomach  Motility secretions
DROMOTROPISM
Trigone sphincter α1 CONTRACTION AV Nodes conduction
activity
GI tract •  Spontaneous
SA, AV Nodes
Salivary glands α1  Secretions depolarization
β1
INOTROPISM
Stomach α1, α2, β2 Motility secretions  Strength of
Atria,
intestines α1, β2 motility
contraction
ventricles
CHROMOTROPISM
LUNGS  Rate of contraction
Tracheal, β2 RELAXATION of
bronchial smooth muscles RELAXATION for FAR
muscles Ciliary muscle
vision
Detrusor RELAXATION
Bronchial glands α1, β2 secretions
Tracheal, RELAXATION of smooth
bronchial muscles
muscles
Bronchial  secretions
glands
intestines  motility
Arteries, CONSTRICTION
arterioles via α1, β2
DILATION
via β2
Veins, venules
β2

Stomach  Motility secretions


DROMOTROPISM
AV Nodes
conduction activity
 Spontaneous
SA, AV Nodes depolarization
INOTROPISM
Strength of
Atria, contraction
ventricles CHROMOTROPISM
 Rate of contraction

Prepared by: MJCN


UIC PharmChem CARES
GREEN REVIEW 2021: Pharmacology I 16

ADRENERGIC AGONISTS
SYMPATHOMIMETIC/ SYMPATHETIC DRUGS
NON- SELECTIVE AGONISTS
→ “UGA FX”
→ Endogenous/ natural catecholamines
→ CATEGORIES:
→ PHARMACODYNAMICS:
o DIRECT ACTING Non selective – ability to o Affinity: β > α
stimulate several/ more than 1 recptors o Very  dose: β
o DIRECT ACTING Selective- ability to stimulate 1 o Rel.  dose: β + α
receptor → PHARMACOKINETICS
o INDIRECT ACTING Releasers- either  or o Absorption:
inhibitors -  oral BA (given IV, SQ, IM, Top. Opth. Soln.),
o CENTRALLY ACTING- CNS acid labile, extensive FPE
- limit their own absorption by causing
vasoconstriction
DIRECT ACTING o Metabolism: MAO and COMT
Non selective Agonists ▪ Dopamine: Homovanillic acid (HVA)
▪ Epi/ NE: Vanillylmandelic acid (VMA)
* Epinephrine/ adrenaline *metabolites assayed in the diagnosis of
* Dipivefrin (+) pheochromocytoma = (
* Norepinephrine metabolites)
* Dopamine → Epinephrine/ adrenaline
→ Dipivefrin
Non selective β agonist
→ Norepinephrine
* Isoproterenol → Dopamine
Selective α1 agonists
(M O P) 1. EPINEPHRINE
→ Aka adrenaline
* Methoxamine
* Oxymetazoline → USES:
* phenylephrine • 1st line in the management of anaphylaxis,
anaphylactic shock, anaphylactoid reaction
Selective α2 agonists
(0.3- 0.5 mg SC q 15-20 mins; 3 doses)
(Met a CLOwn Face in BENZ)
* Methyldopa • Primary cardiac stimulant in ACLS (Advanced
* Clonidine Cardiac Life Support) for px with sudden
* Guanfacine cardiac arrest to have better pumping of heart
* Guanabenz muscle
(1-3 mg IV q 3-5 mins)
Selective β1 agonist
→ Given via IV, SC, TOPICAL OPHTHALMIC
* Dobutamine (1 mg/ml conc. Vial)
Selective β2 agonist → ANAPHYLAXIS
SABA LABA » Most severe form of allergic reaction
(SHORT ACTING β2 AGONSIT) (LONG ACTING β2 AGONIST) » Redness and itching (skin), swelling (mouth)
* Metaproterenol * Formoterol » Excessive release of histamine by mast cells
* Terbutaline * Salmeterol » IgE- mediated
* Pirbuterol * Indacaterol » Not cured by certain antihistamine
Salbutamol/ albuterol * bambuterol → ANAPHYLACTIC SHOCK
TOCOLYTICS » Hypotension (SBP <90 mmHg) that does not
improve with IV fluid administration
* Terbutaline
* Isoxsuprine *normal BP is 120/ 80
* Ritodrine » MGT./TX:
Selective D1 agonist ▪ Inotropropic agent:  strength of
heart’s contraction
* Fenoldopam
▪ Pressors (VASOPRESSORS): anti
INDIRECT ACTING
hypotensive agent;  SBP
Releasers Reuptake Inhibitors
Ephedrine Cocaine
→ ANAPHYLACTOID REACTION
Tricyclic anti depressants
» Similar to anaphylaxis w/ histamine release
Reboxetine (NaRi)
» NOT IgE- mediated
Centrally acting
» More complicated
Phenylpropanolamine Amphetamine → BENEFITS:
phentermine Modafinil ▪ Physiologic antagonist of histamine
phenmetrazine Methyl phenidate
SHORT ACTING
Prepared by: MJCN
UIC PharmChem CARES
GREEN REVIEW 2021: Pharmacology I 17

▪ Stabilizes mast cells (inhibiting release of 2. OXYMETAZOLINE


histamine) → USE:
▪ Mixed with local anesthetics to : • Topical nasal decongestant (spray)
- produce localized vasoconstriction → A/E:
- prolong duration of action RHINITIS MEDICAMENTOSA -Rebound congestion if used > 3
- minimize systemic effects of anesthetics days
*pregnant: spinal anesthetic
▪ Local anesthetics: IV, IM, directly in target SELECTIVE α2 AGONISTS
site
▪ General anesthetics: inhalation, heart
replacement, directly in CNS 1. CLONIDINE
→ Active drug; produces 2 metabolites:
2. DIPIVEFRIN • Apraclonidine
→ Propine® • Brimonidene
→ Prodrug (inactive drug) of epinephrine *Both used for the management of glaucoma
→ Metabolized into epinephrine inside the system → Initially causes (transient) VASOCONSTRICTION; final
→ USE: effect (lasting) is VASODILATION
• Management of open- angle glaucoma → USES:
• Available as 0.1% topical ophthalmic soln. • Management of HTN in hemodialysis px –
3. NOREPINEPHRINE wash out toxic metabolites
→ USES: • Alternative management of ADHD (Attention
• 1st line in the management of septic shock Deficit Hyperactivity Disorder)
caused by infection → A/E:
• Management shock states • Rebound HTN (Clonidine withdrawal
• Alternative in the management of acute heart induced HTN)
failure » Caused by missed doses
→ Admin. via IV infusion (controlled, dextrose type) » TX: immediate institution of oral
Clonidine therapy with Labetalol
4. DOPAMINE (β- blocker)
• Hypertensive Crisis
→ USE: » Severe form of HTN → stroke
• Management shock states associated with » Left unmanaged of HTN
renal failure, acute HF, acutely » TX:
decompensated HF ▪ Na Nitroprusside IV
→ EFFECTS: MOA: sudden drop of BP
• 0.5 -2 mcg/kg/ min: ▪ Nifedipine: CCB- Ca2+
D1 ; renal vasodilation diuresis channel blocker
• 2.5 mcg/kg/ min 2. METHYLDOPA
β1; + chronotropism, + inotropism → A Prodrug
• > 5 mcg/kg/ min → USE:
α1; vasoconstriction • Management of HTN in pregnancy
→ ADVERSE FX: → A/E:
• β1 overstimulation • Sedation
• tachyarrhythmia
•  HR/ Tachycardia • Hepatoxicity (> 2 g/ day dose) usual dose is
→ Admin. via IV 250 mg q 8-12 hrs (500- 750 mg/ day)
→ 2% of pregnant women: false (+) COOMB’s test
» Test for the dx of haemolytic
SELECTIVE α1 AGONISTS anemia
α- methyldopa
1. METHOXAMINE & DOPA decarboxylase
PHENYLEPHRINE α- methyldopamine
→ USES:
• Maintain SBP in hypotensive conditions  Dopamine β- hydroxylase
→ admin. via IV
α-methylnorepinephrine (pharmacologic
→ PHENYLEPHRINE
use)
» admin. orally in combi with other
components
» tx. Nasal congestion
▪ Neozep ® NON SELECTIVE β AGONIST
▪ Decolgen ®
Prepared by: MJCN
UIC PharmChem CARES
GREEN REVIEW 2021: Pharmacology I 18

1. ISOPRETERENOL SELECTIVE D1 AGONIST


→ USES:
• Tx for bronchial asthma as MDI (metered → LOCATION: splanchnic blood vessels in kidneys
dose inhaler)
• 1. FENOLDOPAM
• Not used anymore because of tachyphylaxis → EFFECT: vasodilation
(down regulation of receptors) → USE: management of hypertensive crisis and acute
• Alternative inotropic agent hypotension by normalizing BP
→ Affinity: β2 is stronger ()/ highest affinity INDIRECT ACTING
→ dose= overstimulation of β1 receptors – FATAL Releasers Reuptake Inhibitors
TACHYARRHYTHMIA (HR) Ephedrine Cocaine
SELECTIVE β2 AGONISTS Tricyclic anti depressants
Reboxetine (NaRi)
→ Short duration in the
Centrally acting
system: faster effect
Phenylpropanolamine Amphetamine
→ Given as metered dose
inhalers phentermine Modafinil
→ USE: phenmetrazine Methyl phenidate
SHORT ACTING
• 1st line relievers/
acute relief RELEASERS
• Rescue
1. EPHEDRINE
bronchodilators
→ Dual mechanism:
→ Long duration of action • Indirect - acting: releaser
→ Given orally • Direct- acting: stimulatory fx
→ USE: → From Ephedra sinica (Ma Huang)
• Management of
COPD (Chronic → USES:
LONG ACTING Obstructive • Management of bronchial asthma but is not
Pulmonary induced medical guidelines of hypertension
Disorder) during surgery (IV)
• Controllers/ • Systematically to reduce nasal congestion
maintenance (α1)
→ A/E:
→ USES:
• Tachyarrhythmia
• Induce labor
• Hypertension
• Management of
pre-term labor • Urinary retention
• Management of • Appetite suppression and insomnia
TOCOLYTICS symptomatic
bradycardia
REUPTAKE INHIBITORS
S/SX: difficulty
of breathing and
drowsiness 1. COCAINE
→ used before as local anesthetic
→ drug of abuse (recreational drug)

SABA LABA 2. TRICYCLIC ANTIDEPRESSANTS


(SHORT ACTING β2 (LONG ACTING β2 → antidepression
AGONSIT) AGONIST)
* Metaproterenol 3. REBOXETINE (NaRi)
* Formoterol
* Terbutaline → Noradrenaline reuptake inhibitor
* Salmeterol
* Pirbuterol → Antidepression
* Indacaterol
Salbutamol/ albuterol * bambuterol
TOCOLYTICS CENTRALLY ACTING
* Terbutaline
* Isoxsuprine → Location: CNS
* Ritodrine → Effect: stimulate NE release
→ USES:
• Management of CNS disorders
→ A/E:
• PHENYLPROPANOLAMINE
»  risk of hemorrhagic stroke

Prepared by: MJCN


UIC PharmChem CARES
GREEN REVIEW 2021: Pharmacology I 19

• PPPAMM SELECTIVE α1- BLOCKERS


»  risk of pulmonary HTN → USE:
» Tx: • Management of essential hypotension
▪ Sildenafil (Revatio ®) (unknown cause/ idiopathic)
* Viagra= blue pill • Management of HTN secondary to BPH
(Benign Prostatic Hyperplasia)
1. METHYLPHENIDATE » Non- cancerous breast enlargement
→ Ritalin ® • management of Reynaud’s syndrome
→ 1st line in the management of ADHD
→ Alternative:
» bluish extremities due to cold
• AMPHETAMINE
environment vasospasm
» Not anymore used because of
risk addiction
SELECTIVE α2- BLOCKERS
• TCA → location: Penile cavernosum
• CLONIDINE
1. YOHIMBINE
2. MODAFINIL
→ 1ST line management of Narcolepsy
» Excessive daytime sleeping → TX for erectile dysfunction
→ MOA: Yohimbine blocks α2- receptors (when
3. PHENTERMINE & PHENMETRAZINE activated = vasoconstricted) by vasodilating, blood
ADRENERGIC ANTAGONIST pooling
SYMPATHOLYTIC DRUGS = parasympathetic fx
→ Appetite suppresants DISEASES

ALPHA BLOCKERS BETA BLOCKERS 1. Pheochromocytoma


Non selective α blockers Non selective
→ Tumor hyperplasia in the adrenal medulla (brain)
Phenoxybenzamine Carteolol Penbutolol
→ Hyper secretion of NE and E (NE>E)
Phentolamine Carvedilol Labetalol
→ S/SX:
Pindolol
• Agitation
Selective α1 blockers Cardioselective
• Confusion
Prazosin Bisoprolol Metoprolol
• Paroxysmal HTN
Terazosin Betaxolol Celiprolol
• Tremors
Doxazosin Esmolol nebivolol
• Palpitation
Alfuzosin Acebutol
• Adrenergic sweating (palms and soles)
Tamsulosin
→ DX:
Selective α2 blockers Special Blockers • Clinical signs and symptoms
Yohimbine Celiprolol Acebutol • VMA assay vanillylmandelic acid
Rauwolscine Carteolol Pindolol • CTR and MRI
Labetalol penbutolol
→ TX:
• Initial BP control (α blockers then β)

NON SELECTIVE α- BLOCKERS 2. Carcinoid Syndrome

→ associated with neuroendocrine tumors


1. PHENOXYBENZAMINE → hypertension of 5HT by the enterochromaffin cells
→ Irreversible inhibitor (GIT) where 90% of 5HT come from
→ Anti 5HT and antihistaminic effects → S/SX:
• Flushing
→ Long duration of action • Diarrhea
→ USE: • Severe HA
• Management of carcinoid syndrome → Tx:
• Phenoxybenzamine
2. PHENTOLAMINE • Somastatin analogues (Ocreotide,
Lantreotide)
→ Reversible α- blocker » Have similar effets w/ somastatin
→ USE: (general inhibitory hormone)
• Management of HTN secondary to » Inhibit the release of serotonin
pheochromocytoma
3. Reynaud’s Syndrome

Prepared by: MJCN


UIC PharmChem CARES
GREEN REVIEW 2021: Pharmacology I 20

→ Digital vasospasm (vasoconstricted) in response to RECEPTORS ORGANS EFFECTS


cold environment Skeletal
NM Muscle contraction
→ Caused by exaggerated sensistivity of the blood muscle
vessels to sympathetic stimulation Salivary
Secretions
→ MANAGEMENT: glands
• Cut the fingers Stomach Motility secretions
M1
→ s/sx intestines motility
• pale digestive enzyme
pancreas
• coldhands release
• prone to ulcerations and gangrene as a (-)DROMOTROPISM
result of poor circulation AV Nodes • conduction
activity
→ TX:
SA, AV Spontaneous
• α- blockers
Nodes depolarization
• calcium channel blockers
M2 (-) INOTROPISM
▪ Nifedipine
•  Strength of
▪ Nicardipine Atria, contraction
ventricles (-)CHROMOTROPISM
•  Rate of
CHOLINERGIC RECEPTORS AND THEIR PHYSIOLOGIC
contraction
EFFECTS
All except
heart and
M3
skeletal
EYES muscles
Iris radial M3 Pupillary CONSTRICTION
muscle
Ciliary muscle M3 CONSTRICTION for NEAR BIOSYNTHESIS OF ACETYLCHOLINE
vision
Location: CNS
HEART
AV Nodes (-)DROMOTROPISM • Parasympathetic and sympathetic preganglionic fibers
M2 • conduction activity • Parasympathetic post- ganglionic fibers
SA, AV Nodes M2 Spontaneous
depolarization STEPS:
Atria, M2 (-) INOTROPISM ❶ Active uptake of CHOLINE
ventricles • Strength of
→ Rate limiting step
contraction
→ Inhibited by HEMICHOLINIUMS
(-)CHROMOTROPISM
•  Rate of contraction ❷ single step formation of acetylcholine
❸ Vesicular uptake of AcH
BLOOD VESSELS → Inhibited by VESAMICOL
Arteries, M3  NITRIC OXIDE release ❹ Quantal release of AcH
arterioles and vasodilation → Inhibited by BOTULINUM TOXIN
LUNGS
Tracheal, M3 Bronchoconstriction MANIFESTATIONS OF TOXICITY
bronchial
muscles Diarrhea-motility & secretions
Bronchial M3  secretions Urination- micturition
glands Miosis- constriction pupils
Bradycardia-HR, (-) chrono, ino, dromo
URINARY BLADDER Bronchoconstriction-
Emesis- vomiting-secretions
Detrusor M3 Contraction Lacrimation-  secretion of lacrimal glands
Trigone sphincter M3 relaxation Salivation-  secretion of salivary g.
Sweating- eccrine (sweat all over the body)
GI tract

Salivary glands M1, M3  Secretions

Stomach M1, M3  Motility


secretions

intestines M1, M3 motility

Prepared by: MJCN


UIC PharmChem CARES
GREEN REVIEW 2021: Pharmacology I 21

4. BETHANECOL
DIRECT ACTING
→ Uses:
Choline esters Cholinergic alkaloids • Urinary retention that occurs post operative,
Acetylcholine Pilocarpine or post partum and due to neurogenic atony
Carbachol Muscarine of the bladder
Metacholine Nicotine • GI motility in post surgical distention of the
bethanecol Lobeline abdomen
Varenicline • urine output
Arecoline • intestinal motility (constipation)
INDIRECT ACTING
INTERMEDIATE CHOLINERGIC ALKALOIDS
SHORT ACTING LONG ACTING
ACTING
Edrophonium Neostigmine Malathion
Physostigmine Parathion 1. PILOCARPINE
Pyridostigmine Ecothiophate → Applied topically
Ambenonium Isoflurophate → USES:
demecarium • Tx for open angle glaucoma (mydriasis
DRUGS USED IN THE MGT OF ALZHEIMER’S DISEASE pupil)
Rivastigmine • Mgt. of non obstructive ileus atropine anxiety
Galantamine → ILEUS
Tacrine » movement of intestines = accumulation of
donepezil blockade of food materials and liquids and
intestines
» movement of intestines = accumulation of
A. DIRECT ACTING blockade of food materials and liquids and
→ Receptor agonist intestines
→ Susceptibility to metabolism by AchE » Obstructive ileus: no food, gas, liquids, or
Ach > metacholine > Carbachol > Bethanecol materials can pass through
» Non obstructive: food, ileus and other
*Carbachol and Bethanecol- resistant to AchE
materials can pass through
Choline esters Muscarine (M)

Nicotine (N) Smoking cessation. These


1. ACETYLCHOLINE drugs displace the “being
→ Prototypical cholinergic agonist Lobeline (N) used to nicotine
→ USES: Varenicline(N) consumption” in the body
• Management of glaucoma that they get from the
• Produce brief miosis during eye surgery tobacco. Replacing tobacco
2. CARBACHOL addition w/ these drugs.
→ USES: They displace the nicotine in
• Control  in intraocular pressure after their nicotinic receptors
surgery
→ GLAUCOMA
3. ATROPINE
»  Intraocular pressure →
→ Anticholinergic, UGA
Difficulty in normal flow of aqueous humor
4. ARECOLINE (M, N)
→ dry pupils → blindness
→ BETEL NUT/ NGA-NGA
→ Isopto carbachol
→ Areca catechu
→ Isopto carpine
→ USE:
• Antihelminthic, antiparistic to expel intestinal
3. METACHOLINE
parasitic worms
→ provocholine
→ Uses:
B. INDIRECT ACTING
• Assess bronchial hyperactivity in px who do
→  Ach conc. In the synaptic cleft by inhibiting
not exhibit clinical manifestation
acetylcholinesterase
→ A/E:
• Profound bronchoconstriction in susceptible
individuals when used as diagnostic agent SHORT ACTING
for asthma
• With machine, measurement of lung 1. EDROPHONIUM
capacity, determine the right amount of O2 → CHEMICAL STRUCTURE: Amino alcohol
inhaked ad CO2 exhaled → Effects of AchE: reversible
• If  level = asthma → DOA: <15-30 mins
Prepared by: MJCN
UIC PharmChem CARES
GREEN REVIEW 2021: Pharmacology I 22

→ Uses: → normaly at age 4 y/o - size and activity of thymus


• Diagnosis of myasthenia gravis (TENSILON gland
TEST) → s/sx: progressive muscle weakness
» Autoimmune disorder that causes → onset:
muscle weakness/ paralysis • late afternoon (min by min) ptosis (drooping
• Mgt of neuromuscular blocker toxicity of eyelids)
• late afternoon muscle weakness
INTERMEDIATE ACTING • severe in evening
→ CHEMICAL STRUCTURE: carbamates carbamyl esters → final: diaphragmatic muscle paralysis
→ Effects of AchE: reversible • something is wrong in the diaphragm, bedridden
→ DOA: 2- 8 hrs (3 x a day) (paralysis all over the body)
→ Uses: → diagnosis: clinical s/ sx, imaging of thymoma
• Mgt of atropine toxicity NEOSTIGMINE → tensilon test: edrophonium
• Mgt of neuromuscular blocker toxicity → tx:
NEOSTIGMINE • immunosuppresants prednisone
• Non obstructive ileus PHYSOSTIGMINE • surgical removal
• anticholinesterase
LONG ACTING
→ Pesticides; lipid soluble, crosses BBB
→ NERVE GASES: used for the chemical warfare ANTICHOLINERGIC DRUGS
• Sarin
PARASYMPATHOLYTICS
• Tabon
• Soman- cause drowsiness PROTOTYPE: Atropine
→ CHEMICAL STRUCTURE: organophosphate
M1- Gastric acid secretion
→ Effects of AchE: <24- 48 hrs – potentially reversible
>24-48 hrs- irreversible and needs M2- vagolytic effect (tachycardia)=  HR
antidote
→ DOA: days - weeks M3 – eyes (mydriasis), cyclopegia (loss of accommodation)
→ Uses: ECOTHIOPHATE- mgt of open angle glaucoma
Relaxation/ paralysis of ciliary muscle
to constrict the pupil

DRUGS USED IN THE MANAGEMENT OF ALZHEIMER’S Narrowing of the canal of schlemm


DISEASE
*canal= where aq. Humor flow
o Ach = dementia (memory loss) and disoriented
o Ach – responsible for mental coordination and 
muscle contraction Failure of aq. Humor drainage
o  affinity for AchE active site
o Interact via non covalent bond 
o Can be displaced by excess Ach
intraocular pressure
o Have improved CNS penetration and activity

ALZHEIMER’S DISEASE
Glaucoma
→ Progressive degenerative of the brain that ultimately
results in dementia (mental deterioration) • Bronchi: bronchodilation
→ Exhibit memory loss (recent events) • GIT: ileus constipation
→ Short attention span and disorientation • BLADDER: urinary retention
→ Eventual language loss • EXOCRINE GLANDS: secretions “dry as a bone”
→  Ach precise cause is unknown • ECCRINE GLANDS:
- anhidrosis (lack sweating= HYPERTHERMIA) “hot as
MYASTHENIA GRAVIS hell”
- Flushing- body adjust and produce cutaneous
→ Autoimmune condition vasodilation
→ Antibodies vs Ach receptors (Nm) “ red as beet”
→ Possible association:
Thymoma/ thymic hyperplasia • CNS EFFECTS: agitation, confusion, acute psychosis “mad
» activity of thymus gland as a matter”
» production of T- cell lymphocytes to GENERAL FX:
fight infection
→ larger in children • Restlessness
→ during pubertyin size will be turn into fat • Irritability

Prepared by: MJCN


UIC PharmChem CARES
GREEN REVIEW 2021: Pharmacology I 23

• Hallucinations » Narrowing and


• Antiparkinson fx swelling of bronchioles
• Anitiemetic fx = difficulty in breathing
• Dry mouth Centrally Acting
• acid prod Benztropine
• pancreatic secretory activity (Cogentin®)
• bowel peristalsis Biperiden
Mgt. of Parkinson’s disease
• bladder tone (Akineton®)
Trihexyphenidyl
(Artane ®)
MANAGEMENT OF TOXICITY Scopolamine • Mgt. of motion sickness
• Transdermal patch and in severe
ATROPINE cases
→ Anticholinergic; anti ACh • Combined w/ morphine to induce
→ Uses: twilight sleep during labor:
• Tx. for organophosphate poisoning/ toxicity Twilight sleep= amnesic
ORGANOPHOSPHATE condition during surgery and
» Malathion forgetting what happened after
MOAAch by inhibiting AchE
TX: atropine + enzyme M1 Selective Blocker
regenerators: Pirenzepine Uses:
Pralidoxime (2PAM), diacetyl Telenzepine Adjuncts in mgt. of acid peptic
monoxine (DAM) diseases
ENZYME REGENERATORS:
M3 Selective Blockers
AChE degrade ACh → Ach
• Mydriatic (dilated ophthalmoscopy/ eye Hyoscine- N- Uses:
examination) butylbromide • Antispasmodics/ spasmolytics
• Cycloplegic (lessen eye pain, trauma/ (Buscopan ®) • GI motility disorders
Glycopyrrolate • Renal colic
infection)
Dicycloverine • Sudden pain you get when
• Mgt. of cholinergic agonist toxicity clidinium urinary stones block part of your
• Mgt. symptomatic bradycardia urinary tract
VAGOLYTIC atropine → vagus nerve lysis →
relaxation and regulation of HR or HR • Biliary colic
• Sudden pain occurs due to a gall
• Formulated w/ diphenoxylate to prevent its
stone temporarily blocking the
abuse: cystic duct
Atropine + diphenoxylate → tx: diarrhea • Urinary incontinence
(Lomotil ®) • Involuntary leakage of urine

*DIPHENOXYLATE → opioid antidiarrheal;


lipophilic, crosses BBB; (+) euphoria s/sx

ANTIMUSCARINIC
Mydriatic/ Cyclopegics
Homatropine Uses:
Tropicamide • MYDRIATIC
Cyclopentolate » Dilated ophthalmoscopy/
eye examination to lessen
eye pain
• CYCLOPEGICS
» Relaxation or paralysis of
ciliary muscles

Anticholinergic bronchodilators
Ipatropium Uses:
Tiotropium • 1st line relievers in COPD
Oxitropium (Chronic Obstructive Pulmonary
Disorder)
» Common lung disease
caused by cigarette
smoking
• Alternative in asthma/ bronchial
asthma

Prepared by: MJCN


UIC PharmChem CARES
GREEN REVIEW 2021: Pharmacology I 24

ANTINICOTINICS
MOA: DRUGS: USES:
Inhibit Nn receptors → Hexamethonium Antihypertensive but was
Ganglionic blockers Mecamylamine discontinued due to nonspecific fx
trimethaphan
MOA: Depolarizing/ irreversible/ Non- EFFECTS:
Inhibit Nm receptors → competitive Neuromuscular blockers • Stimulation: tetanic/ continuous
neuromuscular blockers Succinylcholine contraction
• Inhibition: flaccid paralysis
» Neurological condition
characterized by
weakness/ paralysis
and reduced muscle
tone w/o other obvious
cause
A/E:
• diaphragmatic paralysis
• Tremors, myalgia (muscle pain),
myositis (inflam. muscle),
rhabdomyolysis (muscle
weakening), malignant
hyperthermia (TYPE B adr)
Non-depolarizing/ reversible/ USES:
competitive Neuromuscular blockers • Skeletal muscle relaxation during
ISOQUINOLINE: “curium” surgery
Tubocurarine • Mgt. of spastic disorders
Atracurium e.g cerebral palsy
Cistracurium
Miracurium A/E:
• Tubocurarine- anaphylactoid rxn
STEROIDAL: “curonium” Tx: epinephrine
Pancuronium
Rocuronium
vecuronium
CNS

TYPES OF ADR TYPES OF NEUROTRANSMITTERS

Inhibitory

TYPE A Augmented Dose dependent → Inhibit nerve impulses and calm the brian
→ Help create balance
Unpredictable, → Balance mood and are easily depleted when
TYPE B Bizarre
dose dependent the excitatory neurotransmitters are
overactive
Prolong ▪ Excitatory
TYPE C Chronic
treatment → Propagate nerve impulses and stimulate the brain

After years of
TYPE D Delayed COMMON NATURAL NEUROTRANSMITTERS IN BRAIN
treatment
EXCITATORY INHIBITORY
Glutamic acid GABA
Acetylcholine Dopamine
Aspartic acid Serotonin
Norepinephrine (mostly) Glycine

INFLUX OF IONS
(+++) (---)
Depolarized Hyperpolarization
Excitability Relaxation
Overactive CNS Calmness
anxious Depression

Prepared by: MJCN


UIC PharmChem CARES
GREEN REVIEW 2021: Pharmacology I 25

NT ACTION EFFECTS PRECURSOR

• Energy
NE EXCITATORY • Alertness TYROSINE
• Arousal 
• Self esteem DOPA
• Creativity 
DOPAMINE INHIBITORY • Motivation Dopamine
• Attention/ focus 
NE
• Anti- anxiety
• Anti- depression TRYPTOPHAN
SEROTONIN • Appetite- satiety Tryptophan hydroxylase
INHIBITORY
(pineal gland) • Anti-OCD 5-HTP
(5-hydroxytryptophan)
aromatic amino acid  decarboxylase
• sleep 5- HT
(5-hydroxytryptamine aka SEROTONIN)
Serotonin N- acetyl transferase
MELATONIN INHIBITORY N- acetylserotonin
Hydroxyindole-O  methyltransferase (HIOMT)
MELATONIN

• Anti-Irritability GLUTAMATE
GABA • Calms Glutamate  decarboxylase
(gamma- INHIBITORY Excitability GAMMA AMINO BUTYRIC ACID
aminobutyric acid) (GABA)

ANXIETY

→ Body’s natural response to stress


→ Feeling fear or apprehension about what’s to come
→ s/sx:
• mind racing
• sleep disturbances
• breathless/ breathing fast & shallow
• nausea
• appetite loss
• restless
• jelly-like leg
• dizzy, disoriented, lightheaded
• blurry vision
• difficulty in swallowing
• palpitations
• trembling
• sweating/ shivering
→  GABA
→ TX: goal is to GABA (relaxation)
1. BENZODIAZEPINES
- Most widely used anxiolytic
- MOA:  FREQUENCY *frequenZ of
GABA- mediated chloride (-) ion
channel opening

- BENZODIAZEPINE BOOSTERS sits


on GABA receptor

Prepared by: MJCN


UIC PharmChem CARES
GREEN REVIEW 2021: Pharmacology I 26

SHORT 4. ZOLPIDEM (Stilnox ®)


INTERMEDIATE
LONG ACTING ACTING - Type A GABA receptor agonist
(L. A. T. E) - Imidazopyridine structure
(C.O.M.T) - MOA: works by  GABA effects in
Flurazepam Lorazepam (Ativan Clonazepam the CNS by binding to GABA A
(Dalmane®) ®) (Rivotril ® or receptors at the same location as
Klonopin ®) BZDs
Quazepam (Doral Alprazolam (Xanax
®) ® or Xanor ®) Oxazepam (i) TYPES OF ANXIETY
(Serax ®) DESCRIPTION
DISORDER
Diazepam Temazepam
(Valium ®) (Restoril ®) Midazolam Generalized Anxiety Feeling anxious about a
(Dormicum ®) Disorder (GAD) wide variety of things on
Chlorazepate Estazolam
most days over a long
Triamzolam
Chlordiazepoxide period of time (e.g six
(Halcion ®)
(Librium ®) months)

Obsessive Compulsive Ongoing unwanted/ intrusive


Disorder (OCD) thoughts and fears that
DOA: 1- 3 days DOA: 10- 20 hrs DOA: 2-8 hrs cause anxiety (obsession)
and a need to carry out
certain rituals in order to
S/E: CNS depression, drowsiness, dependence addiction, feel less anxious
respiratory depression (+ ethanol, a CNS depressant), (compulsions)
withdrawal symptoms (anxiety, Irritability)
Panic disorder Frequent attacks of intense
feelings of anxiety that
- BENZODIAZEPINE TOXICITY: Antidote: Romazicon® seem like they cannot be
(Flumazenil) brought under control this
may go on to be associated
2. BUSPIRONE
with avoidance of certain
- 5HT1A partial agonist
situations (ex. Going to
- Advantages over BZDs and Barbs:
crowded places)
✓ No potential for abuse
✓ No euphoric effects Social phobia Intense fear of criticism
✓ No dependence bring embarrassed or
✓ No muscle relaxant humiliated even in everyday
3. BARBITURATES situations
- MOADURATION of GABA mediated
chloride ion channel opening (ex. Eating in public or
BARBs bind to BARB receptor (BARB-R) making small talks)

 Specific phobia Fearful feelings about a


particular object or situation
Facilitate GABA + GABA A – R
(ex. Going near an animal,
 flying on a plane or
Prolong the channel opening time (BARBs) receiving an injection)

 Post- traumatic stress Bursts of anxiety any time


disorder from one month after
Increase Cl- current experiencing a traumatic

 (ex. Traumatic delivery,


sexual assault or violence)
hyperpolarization

- BARBITURATES TOXICITY
*structure of barbi is highly acidic
PSYCHOSIS
▪ ANTIDOTES:
sodium bicarbonate → An individual has sensory experience of things that do
(NaHCO3) not exist and / beliefs with no basis in reality
activated charcoal → Distorts a person’s perceptions and thoughts through
(adsorbent) hallucinations and delusions

Prepared by: MJCN


UIC PharmChem CARES
GREEN REVIEW 2021: Pharmacology I 27

NEUROLEPTIC TWO CATEGORIES OF ANTIPSYCHOTICS


FIRST GENERATION/ SECOND GENERATION/
→ A subtype of antipsychotic drug that produces a high CLASSICAL ATYPICAL
incidence of EPS at clinically effective dose or • D2 antagonists • ARE 5HT2A/ D2
catalepsy in laboratory animals • D2 receptor antagonists
→ * EPS blockade • 5HT2A, D4 and
» Extra pyramidal symptoms antipsychotic action weak D2 blockade
» Serious s/e of anti-psychotic drug • Associated with antipsychotic action
» Include acute dyskinesia risk of EPS • Associated with 
- Difficulty in movement • EPS s/e and risk of EPS
Dystonic reactions hyperprolactinemia • Associated with 
- Difficulty in muscle tone due to strong D2 risk of metabolic
blockade- s/e s/e
Akinesia
( dopamine prolactin) • Lesser incidence of
- Absence of muscle movement EPS and
» Two drugs that causes EPS: • Effective against hyperprolactinemia
o Reserpine positive symptoms due too weak D2
blockade
- Anti HTN
• Effective against
- From Rauwolfia serpentina
positive and
- Anti psychotic
negative symptoms
o Chlorpromazine D2
- antipsychotic Location:
GIT = motility
CNS = motor and muscle movement; belief and perception
SCHIZOPHRENIA
regulation
SOCIAL AND OCCUPATIONAL DYSFUNTION:
Positive symptoms: Negative symptoms:
HYPERPROLACTINEMIA
✓ delusions ✓ flat affect
✓ hallucinations ✓ social withdrawal → A condition where you have excess or  prolactin
✓ unusual behaviour ✓ emotional levels in the blood
withdrawal → Prolactin stimulates breastmilk production
Mood disturbances: Cognitive changes
✓ dysphoria ✓ Attention
DOPAMINE
✓ depression ✓ Memory
✓ Executive → Regulation of prolactin production
functioning
→ Restrain the prolactin production
✓ Decision making
→  dopamine  prolactin
POSITIVE AND NEGATIVE SYMPTOMS OF SCHIZOPHRENIA
POSITIVE (+) presence of NEGATIVE (-) absence of ANTIPSYCHOTICS
problematic behaviours healthy behaviour
• Hallucinations • Flat affect (no TYPICAL ATYPICAL
(illusory emotion showing
perceptions), on face LOW POTENCY Aripiprazole (Abilify)
especially auditory) • Reduced social Chlorpromazine (Thorazine) Asenapine (Saphris)
• Delusions (illusory interax. Thioridazine (Mellaril) Clozapine (Clozaril)
beliefs) especially • Anhedonia (no Iloperidone (Fanapt)
persecutory feeling of High Potency Lurasidone (Latuda)
• Disorganized enjoyment) Fluephenazine (Prolixin) Olanzapine (Zyprexa)
thoughts and non • Avolition (less Haloperidol (Halidol) Paliperidone (Invega)
sensical speech motivation, Loxapine (Loxitane) Quetiapine (Seroquel)
• Bizarre behaviours initiative, focus on Risperidone (Risperdal)
Perphenazine (Tritafon)
task)
Thiothixene (Navane) Ziprasidone (Geodon)
• Alogia (speaking
less)
Trifluoperazine (Stelazine)
• Catatonia (moving Pimozide (Drap)
less) Prochlorperazine (Compazine)

TWO NEUROTRANSMITTERS AFFECTING PSYCHOSIS

Inhibitory neurotransmitters:

•  dopamine
•  serotonin

Prepared by: MJCN


UIC PharmChem CARES
GREEN REVIEW 2021: Pharmacology I 28

LOW POTENCY HIGH POTENCY → Spasmodic jerky contractions of group of


(needs higher dose to reach (lowest dose to reach muscles
MEC) MEC) 4. CHOREOATHETOSIS
Chlorpromazine → Occurrence of involuntary movements in
Haloperidol combination of chorea (irregular migrating
Thioridazine
fluphenazine contractions) and athethosis (twisting and
Clozapine
writhing)
• Sedation • EPS 5. TICS STEREOTYPIES
• Orthostatic → Frequently involve in the arms, hands, and
hypotension- sudden entire body
drop of BP → Eyes, face, head
• Anticholinergic fx → More rhythmic and prolonged duration

CHEMICAL CLASSIFICATION OF ATYPICAL HYPOKINESIA


ANTIPSYCHOTICS → Decreased bodily movement
I. Dibenzodiazepine Clozapine → Characterized by partial or complete loss of
(gold standard in muscle movement
the mgt. of
1. AKINESIA
refractory SCZ)
→ Complete loss or impairment of the p;ower
II. Benzisoxazole Risperidone
of voluntary movements
III. Thienobenzodiazepine Olanzapine
IV. Dibenzothiazepine Quetiapine 2. RIGIDITY
V. Benzisothiazolyl Ziprasidone → Inability to be bent or to be forced out of
VI. Quinolinone aripiprazole shape
VII. Benzamide Sulpride,
amisulpride MISCELLANEOUS
1. COMPULSIONS
→ An irresistible urge to behave in a certain
way
2. MANNERISMS
→ Excessive or self-conscious use of
Movement distinctive style in the way of speaking or
disorders
behaving
pyramidal basal ganglia cerebellar 3. AKATHISIA
syndromes disorders disorders → Continual, repetitive rocking motions,
crossing recrossing legs, fidgeting and/
spasticity hyperkinesias hypokinesias miscellaneous ataxia
marching in place

tremor akinesia compulsions

CLINICAL DEPRESSION
choreoathetosis rigidity mannerisms
→ A serious mental disorder that causes persistent
akithisia
dystonia feelings of sadness and loss of interest that interfere
restless legs
with daily life
myoclonus →  5HT  DOP  NE
→ Goal of treatment:  NE  5HT  DOP
tics
stereotypies → Precise cause is unknown
→ Combination of genetics, biology, environment,
psychology
→ MONOAMINE DEFICEINCY THEORY
HYPERKINESIA
→ A disorder marked by hyperactivity and inability to TRICYCLIC ANTI-DEPRESSANTS (TCA)
concentrate

MOA: inhibit re-uptake of NE, serotonin


1. TREMOR PROTOTYPE Amitriptyline, imipramine
→ Involuntary muscle contraction affecting
hands, arms, eyes etc. • Protriptyline • Desipramine
2. DYSTONIA • Amoxepine • Clomipramine
→ Movement disorder in which a person’s • Nortriptyline • trimipramine
muscles contract uncontrollably • Doxepine
3. MYOCLONUS

Prepared by: MJCN


UIC PharmChem CARES
GREEN REVIEW 2021: Pharmacology I 29

SIDE EFFECTS:
SEROTONIN/
• Most common: weight gain NOREPINEPHRINE Venlafaxine
REUPTAKE INHIBITOR
• ECG abnormalities = dysrhythmia/ arrhythmias
(SNRI)
*ELECTROCARDIOGRAM= used to measure heart’s
activity
NORADRENALINE
• Dry mouth, constipation (anticholinergic) Mirtazapine
SELECTIVE SEROTONERGIC
• Sexual dysfunction- loss of libido ANTIDEPRESSANT (NaSSA)

TRAZODONE, NEFAZODONE NORADRENALINE


MOA: 5HT2 antagonist REUPTAKE INHIBITOR Reboxetine
(NaRI)
Withdrawn: severe hepatoxicity and priapism (erection that
does not stop) NORADRENALINE/ Buproprion
DOPAMINE REUPTAKE Wellbutrin
SELECTIVE SEROTONIN REUPTAKE INHIBITOR (SSRI) INHIBITOR (NDRI) Zyban
MOA: inhibits the reuptake/ reabsorption of 5HT

PROTOTYPE fluoxetine

• Sertraline Ser BIPOLAR DISORDER


• Paroxetine Paro
• Fluvoxamine Luv → A mental illness marked by extreme changes in mood
• Citalopram Cita kaya always from high to low and vice versa
• Escitalopram Escited → MANIC = high periods
• Prozac → DEPRESSION = low periods
• zoloft MIXED = feel depressed and elated at the same time
SIDE EFFECTS:
• MANIC MOOD AND BEHAVIOR
• Serotonin withdrawal or discontinuation syndrome
(Serotonin syndrome) • euphoria
→ Tx. Cyproeptadine (5HT blocker) • grandiosity
• Inc. suicidal events • pressured speech
• impulsivity
• excessive libido
MONOAMINE OXIDASE INHIBITOR • recklessness
MOA: inhibits the metabolism/degradation of NTs (NE, 5HT and • social intrusiveness
Dopamine) by inhibiting enzyme MAO • diminished need for sleep
• DYSPHORIC OR NEGATIVE MOOD AND BEHAVIOR
REVERSIBLE • depression
INHIBITOR OF MAO B • anxiety
MAO A BOTH • irritability
(dopamine) • hostility
(5HT, NE) • violence or suicide

Tranylcypromine • PSYCHOTIC SYMPTOMS


Moclobemide Selegiline
Isocarboxazid • delusions
Tx. Parkinson’s
phenelzine • hallucinations
• formal thoughts disorder
• COGNITIVE SYMPTOMS
• Racing thoughts
• distractibility
• disorganization
• inattentiveness

Treatment:

• Lithium
MOA: inhibits directly inositol monophosphate; inhibits
indirectly inositol triphosphate (IP3); alters sodium transport
and may interfere with ion exchange and nerve conduction.
Lithium also enhances the uptake of NE and serotonin
therefore decreasing its action

Prepared by: MJCN


UIC PharmChem CARES
GREEN REVIEW 2021: Pharmacology I 30

IP3 and DAG = skeletal muscle contraction;  overactive/ Alzheimer's Disease


stimulation of CNS
mutation
• Valproic acid (depakene ®) neprilysin hyperphosphorylation of decrease in
resulting in
downregulation tau protein resulting in
• Carbamazepine (tegretol ®) amyloidogenic Ach
neurofibrillary tangles concentration
pathway

amyloid beta neurofibrillary


plaque tangles
ALZHEIMER’S DISEASE accumulation accumulation in
around neurons brain cells
→ Most common cause of DEMENTIA
» Dementia: set of symptoms decreases
cholinergic
characterized by poor memory neurotransmission
and difficulty learning new
information cognitive
impairment
» Caused by damaged brain cells
due to variety of diseases
death
including alzheimer’s
→ Neurodegenerative disorder marked by
cognitive and behavioural impairment that STAGES OF ALZHEMER’S
significantly interferes with social and
Mild
occupational functioning
cognitive Mild Moderate severe
→ It is an incurable diseases with long pre- impairment
clinical period and progressive course
Duration 7 yrs 2 yrs 2 yrs 3 yrs
Origin of Begins in Spreads to Spreads to Spreads
disease medial lateral frontal to
Pathophysiology: temporal temporal lobe occipital
lobe and parietal lobe
◆ Plaques develop in the hippocampus and in
lobe
cerebral cortex
» Hippocampus: structure deep in the
brain that helps encode memories symptoms Short term Reading Poor Visual
» Cerebral cortex: involved in thinking memory problems, judgement, problems
and making decisions loss poor object impulsivity,
» Deposits of protein fragment called recognition, short
beta-amyloid that build up in the poor attention
direction
space of nerve cells
sense
◆ Tangles- twisted fibers of another protein
called Tau protein that build up inside cells

» Kinase transfers PO4 group in tau


protein causing an accumulation of Clinical presentation
neurofibrillary tangles
» Tau protein: helps in regulation of ✓ Memory loss
signal transferring ✓ Aphasia (impairment of language,
affecting production or comprehension
of speech and ability to read or write)
✓ Apraxia ( difficulty with motor planning
Cognitive to perform tasks or movements when
asked, provided that the request/
command is understood and the
individual is willing to perform the test
✓ Agnosia (inability to interpret sensations
and hence recognize things)
✓ Depression
✓ Psychotic symptoms
Non- Cognitive (hallucination and delusions)
Behavioural disturbances

Inability to care for self (dressing, bathing,


Functional toileting, eating)

Prepared by: MJCN


UIC PharmChem CARES
GREEN REVIEW 2021: Pharmacology I 31

MOA: works by regukating glutamate, an important


brain chemical; inhibit NMDA- N-methyl-D-aspartate
Diagnosis:
receptor- binding to glutamate
• No specific test Drug: Memantine
• Complete accuracy only after death when
microscopic examination of the brain reveals Glutamate: when produced in excessive amounts,
the characteristic plaques and tangles glutamate may lead to brain cell death
• Physical and neurological exam:
Check overall neurological health by testing
reflexes, muscle tone, and strength, ability to
get up from a chair and walk across the Cognitive enhancers:
room, sense of sight and hearing,
coordination and balance Donepezil Mild to Tablet, 5 mg- 4 wks:
(Aricept) severe orally 23 mg 5-10 mg
• Laboratory test:
disintegra- QD 3mos:
Blood test may help rule out other potential ting tablet 10-23
causes to memory loss and confusion such mg
as thyroid disorders or vitamin deficiencies Galantamine Mild to Tablet, oral 4 mg to 4 wks
• Mental status and neuropsychological (Razadyne) moderate soln, 12 mg
testing BID
Mental status test to assess memory and
other thinking skills. Doctor may suggest a extended 8 mg to 4 wks
more extensive assessment of thinking and release 24 mg
memory. Longer forms of capsule ER QID
neuropsychological testing may provide Rivastigmin Mild to Capsule 1.5 mg- 2 wks
additional details about mental function e (Exelon) moderate 6mg
compared with others’ similar age and BID
educational level Patch 4.6 mg- 4 wks
9.5 mg
• Brain imaging:
QID
» MRI (magnetic resonance imaging)- assess
Memantine Mod- Tablet, oral 5 mg-10 1 wk
whether shrinkage in brain (atrophy) regions (Namenda) severe soln mg QID
implicated in alzheimer’s disease has
occurred
» CT (computerized tomography) Side Effects:
Produces cross-sectional images of brain
Cholinesterase inhibitors:
used chiefly to rule out tumors, strokes and
head injuries ✓ Nausea
» PET (positron emission tomography) ✓ Vomiting
Able to detect brain level of plaques and ✓ Diarrhea
tangles the two hallmark abnormalities linked ✓ Wt. loss
to alzheimer’s ✓ Loss of appetite
✓ Muscle weakness
✓ Vivid dreams/ nightmares (donepezil)
Treatment: ✓ Syncope
✓ Bradycardia
Primary goal: to symptomatically treat cognitive difficulties and ✓ Arrhythmias
preserve patient functioning as long as possible ✓ Sinoatrial block
✓ anorexia

Secondary goal: treating the psychiatric and behavioural


sequelae that occur as a result of disease NMDA receptor antagonist:

✓ Dizziness
Pharmacological therapy ✓ HA
✓ Constipation
1. Cholinesterase inhibitors ✓ Confusion
MOA: prevent the breakdown of Ach, a brain chemical ✓ Hallucinations
believed to be important for memory, thinking by
inhibiting Acetylcholinesterase
Drugs: Donepezil, rivastigmine, galantamine Managing behaviors
2. Anti-glutamatergic therapy
→ Help with depression, aggression, restlessness, and
anxiety
Prepared by: MJCN
UIC PharmChem CARES
GREEN REVIEW 2021: Pharmacology I 32

• Citalopram (celexa) o Occurs most often at rest


• Sertraline (Zoloft) o May involve diaphragm, tongue, lips and jaw
• Duloxetine (Cymbalta) o Increases with stress
• Mirtazapine (remeron) ◆ Bradykinesia:
• Bupropion (wellutrin) o Loss of normal arm swing while walking
• Imipramine (tofranil) o  blinking of eyelids
o Loss of ability to swallow
o Blank expression
PARKINSON’S DISEASE o Difficulty initiating movement
◆ Muscle rigidity
→ Usual age at time of diagnosis: 55-65 y/o o  resistance to passive movement
→ Higher incidence is reported among males o Cog wheel, jerky slow movement
Drug- induced parkinsonism

→ Parkinson- like symptoms 4 Motor Hallmarks:


→ (MPPP) 1- methyl-4 phenyl-4-propionoxypiperidine or
demethylprodine: opioid analgesic 1. Tremor
→ Toxic impurity: (MPTP) 1-Methyl-4-phenyl-1,2,3,6- ◆ Involuntary shakiness at rest
tetrahydropyridine: parkinsonism ◆ Resting tremor (sole chief complaint)
2. Rigidity
Pathophysiology: ◆ Stiffness
◆ Death of dopaminergic neuron in substancia ◆  muscle tone and resistance to movement
nigra (black substance) due to continuous muscular contractions
» Substancia nigra: located in both 3. Bradykinesia
sides of brain (1 in each side of the ◆ Slow movement
midbrain) Hypokinesia
» Part of basal ganglia (controls ◆ Lessened movement
movement that connects to motor Akinesia
cortex)
» Pars reticulate: receives signals ◆ Absence of movement
from striatum (caudate, putamen)
Micrographia
and relays signal to thalamus via
neurons rich in GABA ◆ Features abnormally small, cramped
» Pars compacta: affected; sends handwriting or the progression to
messages to striatum via neurons progressively
rich in dopamine Nigostriatal
pathway (helps stimulate cerebral 4. Postural instability
cortex) ◆ Problems w/ balance
◆ Can lead to falls
Patient w/ no neurons/ death of neurons
Diagnosis

◆ Presence of bradykinesia and at least one of the ff:
Pars compacta
✓ Resting tremor
 ✓ Rigidity
✓ Postural instability
Movement
◆ Exclude other types of parkinsonism or tremor
 disorders
◆ Presence of at least three supportive positive criteria
Hypokinesia (slow movements) ✓ Asymmetry of motor signs and symptoms
✓ Unilateral onset
◆ Loss of neurons (depigmentation),
✓ Progressive disorder
substancia nigra specifically pars compacta
✓ Resting tremor
◆ Presence of lewy bodies in microscopic
✓ Excellent response to carbidopa/ L-DOPA
examination
✓ L-DOPA response for five years or longer
◆  dopamine
✓ Presence of L-DOPA dyskinesias

Differential diagnosis:
Signs and symptoms:
To rule out any other diseases/ diorders causing tremor
◆ Tremor: problems not related to parkinson’s disease
o Commonly in arms and hands
o Pill rolling motions w/ fingers
Prepared by: MJCN
UIC PharmChem CARES
GREEN REVIEW 2021: Pharmacology I 33

Treatment: DOSE Starting: 0.5 mg/ day


Maintenance: 1- 6 mg/ day
Desired outcomes: D/I Potassium products:  GI motility (therefore take
✓ To improve motor and non-motor symptoms so that with food)
A/E Anticholinergic fx, confusion, sedation
patients are able to maintain the best possible quality
of life
✓ Preservation of ability to perform activities of daily 2. Trihexyphenidyl (artane ®)
living DF tablet
✓ Improvement of mobility DOSE Starting: 1-2 mg/ day
✓ Minimization of a/e, tx compications Maintenance: 6- 15 mg/ day
✓ Improvement of non-motor features- cognitive D/I Potassium products:  GI motility (therefore take
impairment, depression, fatigue and sleep disorders with food)
A/E Anticholinergic fx, confusion, sedation)
Non pharmacologic therapy

◆ Deep brain stimulation/ bilateral, chronic, high Amantadine (symmetrel ®)


frequency electrical stimulation MOA Precisely unknown, but enhancement of DA
◆ Extension release from pre-synaptic terminals and inhibition
◆ Neurostimulator of glutamatergic N-methyl- D-aspartate receptors
are implicated
DF capsule
Pharmacologic therapy DOSE Starting: 100 mg/ day
Maintenance: 200-300 mg/day
1. Carbidopa/ levodopa (sinemet ®) D/I Anticholinergics: additive effects; blurred vision,
2. Anticholinergic medications urinary retention, constipation
a. Benztropine A/E Confusion, livedo reticularis; reversible condition
b. Trihexyphenidyl characterized by diffuse mottling of the skin
3. Amantadine affecting the upper or lower extremities and often
4. Monoamine oxidase B inhibitor accompanied by lower extremities edema
a. Selegiline
b. Rasagiline MAO B INHIBITORS
5. Catechol-O-methyltransferase inhibitors MOA Helps to block the breakdown of DA in the brain
a. Entacaopne MAO A → 5HT and NE
b. Tolcapone MAO B → DA →  DA
6. Dopamine agonist
a. Apomorphine
b. Bromocriptine 1. Selegiline (eldepryl ®)
c. Pramipexole DF Tablet, ODT (zelapar ®)
d. Ropinirole DOSE Starting: 5-10 mg/ day
Maintenance: 5-10 mg/ day
e. Rotigotine
D/I Carbamazepine/ Oxycarbazapine: risk for
dangerous  BP and seizures
Carbidopa/ levodopa (sinemet ®) Dextromethorphan: cause psychotic episodes or
MOA Administration of dopamine is ineffective because bizarre behaviour
it does not cross BBB Anticholinergics: additive s/e: blurred vision,
Levodopa- metabolic precursor of DA, crosses urinary retention, constipation
BBB, converts DA in the brain MAOis: additive effects, risk for  BP and
DF tablet seizures
DOSE Starting: 100-300 mg/ day A/E Confusions, insomnia, hallucinations, orthostatic
Maintenance: 300-1000 hypotension
D/I MAOis: Risk of heart effects
Iron products:  carbidopa/ levodopa levels
2. Rasagiline (azilect ®)
Phenytoin:  beneficial effects of carbi/levo
DF
A/E Drowsiness, dyskinesia, nausea (taken w/ food)
DOSE Starting: 0.5 -1 mg/ day
Maintenance: 0.5- 1 mg/ day
ANTICHOLINERGIC MEDICATIONS D/I Carbamazepine/ Oxycarbazapine: risk for
MOA Administration of dopamine is ineffective because dangerous  BP and seizures
it does not cross BBB Dextromethorphan: cause psychotic episodes or
Levodopa- metabolic precursor of DA, crosses bizarre behaviour
BBB, converts DA in the brain Anticholinergics: additive s/e: blurred vision,
urinary retention, constipation
MAOis: additive effects, risk for  BP and
1. Benztropine (Cogentin ®) seizures
DF Tablet, vial A/E Nausea- take w/ food
Prepared by: MJCN
UIC PharmChem CARES
GREEN REVIEW 2021: Pharmacology I 34

Maintenance: 1.5- 4.5 mg/ day


D/I DA antagonists
CATECHOL-O-METHYLTRANSFERASE A/E Confusions, drowsiness, hallucinations/
MOA  perioheral conversion of L-DOPA to DA thus delusions, impulsivity, orthostatic hypotension
enhancing central L-DOPA BA

3. Ropinirole (Requip ®)
1. Entacapone (Comtan ®) DF Tablets, requip XL
DF caplet DOSE Starting: 0.75 mg/ day
DOSE Starting: 200-600 mg/ day Maintenance: 9.24 mg/ day
Maintenance: 200-1600 mg/ day D/I DA antagonists: reduce efficiency of ropinirole
D/I  HR and excessive changes in BP: estrogens
Isoproterenol A/E Confusions, drowsiness, hallucinations/ delusions
Dobutamine
Methldopa
Dopamine 4. Rotigitine (Neupro ®)
NE DF
Bitolterol DOSE Starting: 2 mg/ day
isoetherine Maintenance: 2- 8 mg/ day
A/E Confusions, insomnia, hallucinations, orthostatic D/I DA antagonists
hypotension A/E Confusions, drowsiness, hallucinations/ delusions

2. Tolcapone (Tasmar ®) EPILEPSY


DF caplet
DOSE Starting: 300 mg/ day → Chronic disorder characterized by recurrent seizures
Maintenance: 300-600 mg/ day → CONVULSIONS: abnormal/ involuntary muscle
D/I  HR and excessive changes in BP: contractions/ jerky movements
Isoproterenol → SEIZURES: sudden excessive abnormal discharge of
Dobutamine cerebral neurons; abnormal or neuronal activity
Methldopa → Causes:
Dopamine ✓ Heredity
NE
✓ Infection, neoplasm (new growth of
Bitolterol
abnormal tissue), head injury
isoetherine
✓ Environmental- alteration in blood gases, pH,
A/E Augmentation of L-DOPA side effects, diarrhea,
liver toxicity electrolytes, glucose availability

Hallmarks
DOPAMINE AGONISTS 1. Hyperexcitability
MOA Stimulate post-synaptic dopamine receptors → Abnormal responsiveness of a neuron to an
directly more dopamine released
excitatory input fires multiple discharges
instead of usual one/ two
1. Apomorphine (Apokyn ®) 2. Hypersynchrony
DF → Recruitment of large numbers of neighboring
DOSE Starting: 1-3 mg/ day neurons into an abnormal firing mode
Maintenance: 3-12 mg/ day
Pathophysiology
D/I 5HT3 antagonists: may cause hypotension and
loss of consciousness ◆ Increased CNS excitability
A/E Drowsiness, nausea, orthostatic hypotension o Membrane depolarization
o Increased excitatory input
2. Bromocriptine (Panodel ®) o  inhibitory GABA input
DF tablet
DOSE Starting: 2.5 -5 mg/ day
Maintenance: 15-40 mg/ day
D/I Triptans
DA antagonists:  efficiency of bromocriptine
Ergot alkaloids:  s/e of ergot alkaloids
Grapefruit products:  serum concentration of
bromo

3. Pramipexole (Mirapex ®)
DF Tablet, ER
DOSE Starting: 0.125 mg/ day

Prepared by: MJCN


UIC PharmChem CARES
GREEN REVIEW 2021: Pharmacology I 35

GENERALIZED SEIZURES
pre-
synapse → Electrical discharge spread to both hemisphere
→ May be convulsive (shaken repeatedly) or non-
post convulsive (absence)
synapse → Immediate loss of consciousness occurs
→ Types:
major • Generalized tonic-clonic (grand mal) seizures
major inhibitory
excitatory NT NT • Absence (petit mal) seizures
• Tonic seizures
• Atonic seizures
glutamate GABA • Clonic and myoclonic seizures
• Febrile seizures
◆ Generalized tonic-clonic (grand mal) seizures
NMDA influx of Cl-
ions → Characterized by:
TONIC PHASE: continuous rigidity of all
extremities; less than minute; abrupt loss of
entry of Na+ hyperpolarization consciousness, respiration arrest
and Ca2+
CLONIC PHASE: massive jerking of body
(rapid contraction and relaxation); 2-3 mins;
inhibitory lip/ tongue biting, fecal and urinary
depolarized
incontinence
→ Tongue or cheek bitten, urinary incontinence
excitatory relaxation common
→ < 5 mins= not emergency
→ Seizure followed by period of confusion and
exhaustion due to depletion of energy
FOCAL SEIZURES ◆ Absence seizure
→ Typical absence seizures consists if staring
→ Involve only a portion of the brain for few seconds (altered consciousness)
→ One lobe of one hemisphere then returning to full function as if nothing
PARTIAL SEIZURES occurred
→ Brief duration < 10 seconds
◆ Simple Partial Seizures → Mild clonic jerking of eyelids, patient stares
→ Without loss of consciousness (preserved) and exhibit rapid eye blinking
→ Confined to a single locus in brain → Begin in childhood or adolescence and may
→ Abnormal activity in one limb/muscles occur up to hundreds of times a day
→ Without autonomic symptoms (nausea, BP → Patient has no recollection of events
changes) ◆ Myoclonic seizures
→ Convulsive jerking, paraesthesia, psychotic → Seen in generalized tonic-clonic seizures,
symptoms, autonomic dysfunction partial seizures, absence seizures, infantile
◆ Complex Partial Seizures spasms
→ With loss of consciousness → Short episode of muscle contractions
→ Impaired consciousness from the onset → Occurs due to permanent neurologic damage
→ Exhibits complex sensory hallucination due to hypoxia, uremia, encephalitis or drug
→ Motor dysfunction may involve chewing poisoning
movements, diarrhea, or urination o Hypoxia: loss of oxygen
o Uremia: uric acid in the blood
o Encephalitis: inflammation in brain/
drug poisoning
Partial
seizure → Single or multiple myoclonic muscle jerks
◆ Atonic seizures
→ Sudden loss of postural tone, if standing
simple partial - remains
conscious posture falls suddenly and may be injured, if
seated, may suddenly drop forward
aura
→ Impaired consciousness (abrupt onset and
complex partial- loss
consciousness brief), loss of postural tone, or enuresis,
begin in childhood and usually cease by age
Secondarily generalized seizure 20 years
generalized seizure- tonic clonic
w/ unconsciousness

Prepared by: MJCN


UIC PharmChem CARES
GREEN REVIEW 2021: Pharmacology I 36

Status epilepticus • Teratogenic: affects/ cause s/e to fetus


(hands and feet)
→ Two or more seizures occur without recovery or full
consciousness in between episodes
→ These may be focal or primary generalized convulsive 2. Carbamazepine (Tegretol ®)
or non- convulsive Use Doc for generalized tonic- clonic seizures, partial
→ Life-threatening and requires emergency treatment seizures, prolonged therapy for Status
usually consisting of administration of a fast ating epilepticus
medication such as BZD (IV) followed by slower A/E Diplopia
acting medication such as Phenytoin Cognitive dysfunction
Drowsiness
→ Lasts more than 30 mins
Ataxia
Treatment: Blood dyscrasias
SJS (rare)
1. Na+ channel blockers Enzyme inducer
a. Phenytoin Craniofacial anomalies
b. Carbamazepine Spina bifida
c. Lamotrigine Rare s/e • Diplopia: double vision/ “duling”
d. Lacosamide • SJS (stevens Johnson syndrome): rare
e. Topiramate but serious s/e to a medicine/ drug
f. Zonisamide • Affects the mucous membranes and
blisters from inside the skin making it
hard to eat, swallow or even pee
2. Ca2+ channel blockers
• Genetics-based
a. Ethosuxamide
b. Gabapentin
3. GABA modulators 3. Lamotrigine
a. Tigabine A/E Dizziness, rash, ataxia, SJS, nausea
b. Vigabatrin
c. Gabapentin
4. Others Ca2+ CHANNEL BLOCKERS
a. Valproic acid MOA 4 Types of Ca2+ channels:
• P-TYPE “purkinje”- smooth muscles,
b. Felbamate
blood vessels
Na+ CHANNEL BLOCKERS • N-TYPE “neuronal”- neurons, CNS
MOA Binds to sodium channels, stabilize its inactive • T-TYPE “Transient”- neurons, CNS
conformation and reduces excitability of • R-TYPE “Residual”- smooth muscles
neurons Drugs Ethosuxamide
Gabapentin
Glutamate → NMDA receptor → open Ca2+,
Na+ channel → +++ → excitatory
DRUGS Phenytoin (Dilantin®) 1. Ethosuxamide
Carbamazepine (Tegretol ®) Moa Blocks low threshold T-type calcium channels
Lamotrigine Use Doc for absence seizures (preferred for children)
Lacosamide a/e GI distress
Topiramate Lethargy
Zonisamide HA
Behavioural changes

1. Phenytoin (Dilantin®) 2. Gabapentin


Use Doc for generalized tonic- clonic seizures, partial Moa Inhibits N-type calcium channel to decrease
seizures, prolonged therapy for Status glutamate release
epilepticus
Rare s/e • Nystagmus: vision condition in w/c the  glutamate → NMDA receptor → excitatory
eyes make repetitive movements often
result in reduced vision affecting
GABA MODULATORS
balance and coordination
Drugs Tigabine
• Hirsutism: excessive body hair in men
Vigabatrin
and women on parts of the body where
Gabapentin
hair is normally absent/ minimal
• Fetal hydantoin syndrome: upper facial
features, upturned nose, mild midfacial 1. Tigabine
hypoplasia, long upper lip within MOA Inhibits GABA transporter (GAT-1) prolonging the
vermillion border, lower distal action of neurotransmitter; increase GABA in
hypoplasia synaptic cleft

Prepared by: MJCN


UIC PharmChem CARES
GREEN REVIEW 2021: Pharmacology I 37

Back up Levetiracetam
Topiramate
2. Vigabatrin zonisamide
MOA Irreversible inhibition of aminotransferase (GABA- Status epilepticus
T) that degrades GABA; Increase concentratiom Diazepam (IV) or lorazepam- stop attacks & short term
in synaptic cleft control
Phenytoin- prolonged therapy
General anesthesia- patient remains unresponsive
3. Gabapentin
MOA - Structurally similar to GABA
- Increase GABA release into synapses
- Same action as GABA

GASTROINTESTINAL SYSTEM

OTHERS Gastrointestinal Drugs


Drugs Valproic acid disorders
Felbamate Motility promoters Metoclopramide
Levetiracetam cholinomimetics
Irritable bowel Anticholinergic (dicycloverine)
syndrome Serotonin antagonists
1. Valproic acid Inflammatory bowel 5-ASA
MOA Blocks sodium channels and NMDA glutamate disease Corticosteroids immunosuppressant
channels;  excitation response Anti-TNF drugs
Rare Spina bifida: birth defects that occurs when the
s/e spine and spinal cord don’t form properly Antimimetics 5-HT3 blockers (ondansetron)
D2 blockers (prochlorperazine)
H1 blockers (diphenhydramine)
2. Felbamate Antimuscarinics (scopolamine)
MOA Blocks glutamate NMDA receptors Corticosteroids (dexamethasone)
a/e Aplastic anemia, hepatic failure Cannabinoids (dronabinol)
Neurokinin receptor antagonists
(aprepitant)
3. Levetiracetam Acid peptic disease Antacids
MOA Modulates synaptic neurotransmitter release H2 blockers (cimetidine)
through binding to synaptic vessel protein SV2A Proton pump inhibitors (omeprazole)
A/E Dizziness, sedation, weakness, irritability, Mucosal protective agents
hallucinations (sucralfate, misoprostol, bismuth)
Clinical Applications Antibiotics
Generalized tonic-clonic seizures Others Pancreatic lipase
Doc Valproic acid Laxatives
Carbamazepine Antidiarrheal ursodiol
Phenytoin
Alternatives Phenobarbital
Lamotrigine
topiramate PEPTIC ULCER DISEASE
Partial seizures
Duodenal ulcers
Doc Carbamazepine
Lamotrigine → Pain relieved by meals
phenytoin → Occurs 2-3 hrs after meals
Alternatives Felbamate
→ Most common
Phenobarbital
→ MELENA occurs
Topiramate
Valproic acid o Melena: black tarry stools and offensive
Absence seizures smell
Doc Ethosuxamide o Difficult to flush away due to alteration and
Valproic acid- useful in px with degradation of blood by intestinal enzymes
concomitant generalized tonic clonic or
Gastric/ stomach ulcers
myoclonic seizures
Alternatives Clonazepam → Pain increased by meal
Lamotrigine → Occur 30 mins to 1 hr after meal
levetiracetam
→ Not as common
Myoclonic seizures
→ Vomiting occurs
Doc Valproic acid
Lamotrigine- adjunctive use Drugs
Clonazepam- may be effective

Prepared by: MJCN


UIC PharmChem CARES
GREEN REVIEW 2021: Pharmacology I 38

1. Antacids 3. Famotidine (Pepcid ®)


a. Sodium bicarbonate Moa Most potent; most BA
b. Calcium carbonate
c. Aluminium hydroxide
d. Magnesium hydroxide 3. Nizatidine (axid ®)
Moa Newest, non-hepatotoxic
2. H2 receptor antagonist/ H2 blockers
a. Cimetidine
b. Ranitidine PROTON PUMP INHIBITOR
c. Famotidine Moa Binds covalently with H/K/ATPase enzyme; reduces
d. Nizatidine 90-98% acid
3. Proton pump inhibitors Note Acid labile; given 1 hr before food
a. Omeprazole Drugs Omeprazole (Prilosec ®) (losec ®)
b. Lansoprazole Lansoprazole (prevacid ®)
c. Rabeprazole Rabeprazole (pariet ®)
d. Pantoprazole Pantoprazole (protonix ®)
e. Esomeprazole Esomeprazole (nexium ®)
4. GI protectant
a. Sucralfate
b. Misoprostol GI PROTECTANT
c. Colloidal bismuth
5. Antibiotics
1. Sucralfate (aluminium sucrose sulphate)
ANTACIDS Moa Polymerizes in acidic environment w/c binds to
ulcers, accelerates healing and reduces recurrence
1. Sodium bicarbonate
Moa Reacts rapidly producing CO2, absorbed in gut 2. Misoprostol (cytotec ®)
A/E Belching, flatulence, metabolic alkalosis, rebound Moa PGE 1 analog; stimulates mucus and bicarbonate
acidity, fluid retention secretion
a/e teratogenic
2. Calcium carbonate
Moa Reacts more slowly, absorbed in gut 3. Colloidal bismuth
A/E Belching, flatulence, metabolic alkalosis, kidney Moa Coats ulcer, stimulate PGE fx, direct antimicrobial
stones fx
Drugs Bismuth subsalicylate- used in diarrhea,  stool
frequency and liquidity
3. Aluminum hydroxide Bismuth subcitrate potassium
Moa No carbon dioxide produced, recats slowly, not
significantly absorbed
A/E constipation ANTIBIOTICS
Moa Used in ulcers caused by H. pylori
Drug regimens
4. Magnesium hydroxide
PPI+ clarithromycin+ amoxicillin/ metronidazole (BID)
Moa No CO2, slow, not significantly absorbed
PPI (BID) + Bismuth subsalicylate+ tetracycline+
A/E diarrhea metronidazole (QID)
PPI (BID) + bismuth subsalicylate + tetracycline +
H2 RECEPTOR ANTAGONIST metronidazole
Moa Inhibits 60-70%
Histamine: most potent acid stimulus for acid CONSTIPATION
secretion
s/e Diarrhea, constipation, myalgia →  in frequency of fecal elimination
→ Treatment: laxatives
1. Bulk forming laxatives
a. Psyllium
1. Cimetidine (tagamet ®) b. Methylcellulose
Moa Enzyme inhibitor, blocks histamine receptor c. Polycarbophil
A/E Gynecomastia: breast enlargement impotence: 2. Saline & osmotic laxatives
erectile dysfunction a. Sodium and magnesium salts
Galactorrhea: milk discharge b. Glycerine
c. Lactulose
3. Stimulant laxative
2. Ranitidine (zantac ®)
a. Sennosides, cascara
b. Biascodyl
Prepared by: MJCN
UIC PharmChem CARES
GREEN REVIEW 2021: Pharmacology I 39

c. Castor oil
4. Emollient laxatives
a. Glycerine spp OPIOID AGONISTS
Moa Those that do not cross BBB and no analgesic fx
b. Mineral oil
Drugs Loperamide (Imodium®)
c. Docusate Na
diphenoxylate
5. Chloride channel activator
a. Lubiprostone
6. Opioid antagonist BILE ACID BINDING RESINS
a. Methylnaltrexone Moa Binds to bile salts to decrease diarrhea
b. Alvimopan s/e Bloating, flatulence, constipation
Drugs Cholestyramine (questran®)
BULK FORMING LAXATIVES Colestipol (colestid ®)
Moa Distension of colon; stimulate peristalsis Colesevelam (welchol ®)
Drugs Psyllium
Methylcellulose
Polycarbophil SOMASTOSTATIN
Effects ✓ Inhibits secretion of gastrin, glycogen,
GH, insulin, secretin, PP, 5-HT
✓  intestinal fluid secretion
SALINE AND OSMOTIC LAXATIVES ✓ Slow GI motility, inhibits GB contraction
Moa  stool liquidity by increasing concentration ✓ Inhibits secretion of anterior pituitary
throughout colon hormone
Drugs Sodium and magnesium salts- should not be taken
in px with HTN, CHF, renal impairment
Glycerine ANTI-EMETICS
Lactulose
5-HT3 ANTAGONISTS
Indications Chemotherapy-induced nausea and vomiting;
STIMULANT LAXATIVES given 30 mins before chemotherapy; post-
Moa Stimulate GIT movements operative and post- radiation nausea and
Drugs Sennosides, cascara- chronic use can lead to vomiting
cathartic clon (not be used > 1 week) Drugs Ondansetron (ondem ®): 32 mg/day.k
Biascodyl (dulcolax ®) Granisetron (graniset®): 10 mcg/kg/day
Castor oil- ricinoleic acid= irritatant; CI to pregnant Dolasetron: 1.8 mg/ kg/day
women Palonosetron: 250 mcg by slow IV injection
30 mins before chemo

EMOLLIENT LAXATIVES
Moa Permits water and lipid to penetrate stool softening H1 BLOCKERS
materials Indications Most effective drugs for motion sickness
Drugs Glycerine spp Drugs Meclizine (bonamine ®)
Mineral oil Cyclizine (marezine®)
Docusate Na Dimenhydrinate (Dramamine ®)
Diphenhydramine (Benadryl®)
Promethazine- used by pregnant, NASA
CHLORIDE CHANNEL ACTIVATOR

MUSCARINIC ANTAGONIST
1. Lubiprostone (amitiza ®) Indications Anticholinergic
Moa Stimulate type 2 Cl channels in the small intestine, Drugs Scopolamine (hyoscine ®)- transdermal
 Cl- rich fluid secretion, stimulates motility patch for motion sickness

OPIOID ANTAGONISTS CANNABINOIDS


Moa Tx for opioid induced constipation, post-operative Moa Stimulates CB1 subtype of cannabinoid
ileus; water absorption receptor present in neurons around
Opioids cause constipation,  GI motility, fecal vomiting centre
Drugs Methylnaltrexone Drugs Dronabinol (marinol, syndros)- adjuvant in
Alvimopan chemo-induced vomiting; psychoactive
substance
Nabilone (Cesamet ®)
DIARRHEA * Analogues of delta-9 TH, an active
principle of Cannabis sativa
ADSORBENTS
Moa Adsorbs water from stool
Drugs Kaolin: hydrated MgAlSiO4
Pectin: indigestible carbohydrate

Prepared by: MJCN


UIC PharmChem CARES
GREEN REVIEW 2021: Pharmacology I 40

DOPAMINE ANTAGONIST
Moa Antagonize D2 receptors in chemoreceptor
trigger zone Anesthetic
Drugs Metoclopramide (reglan ®)
- 2.5 mg BID
general local
- Oral and IV
- Crosses BBB
Domperidone (Motilium ®) intravenous inhalational esters amides
- 10 mg BID
- oral halogented
gas procaine lidocaine
- don’t cross BBB hydrocarbon
*both are prokinetic agents 5HT4 agonist
nitrous
halothane cocaine prilocaine
oxide
INFLAMMATORY BOWEL DISEASE

Ulcerative colitis

→ Typically begins in the rectum and may extend to Pre-anesthetics/ Anesthetic Adjuncts
involve the entire colon
→ Usually affects only the inner layer of the bowel wall Functions:

Crohn Disease ✓ Relieves anxiety Benzodiazepine


✓ Prevent gastric acid secretion H2 blockers
→ Most commonly involves the end of the small ✓ Prevent allergic reactions Antihistamine
intestine and beginning of the colon and may affect ✓ Prevent aspiration of stomach contents and post
any part of the GI tract in patchy pattern surgical nausea and vomiting anti-emetics
→ May affect all layers of bowel wall ✓ Prevent bradycardia and secretion of fluids into
respiratory tract anticholinergic drugs
AMINOSALICYLATES
✓ Facilitate intubation and relaxation neuromuscular
Moa Regulates inflammatory process by
decreasing pro-inflammatory cytokines such blocking agents
as IL (interleukin), TNF (tumor necrosis ✓ Provide analgesia opioids
factor),  COX2 activity w/c PG formation
Drugs Mesalamine: generic 5-ASA formulation 1. General anesthesia
Pentasa ®: mesalamine that contains time- 2. Inhalation anesthesia
released microgranule 3. Intravenous anesthesia
Asacol ®: mesalamine coated in pH
sensitive resin that dissolves at pH 7
Lialda ®: uses pH-dependent resin that
GENERAL ANESTHESIA
encases a multimatrix core
Rowasa®: 5-ASA enema → Reversible state of central nervous system depression
Canasa ®: 5-ASA suppository → Causes loss of response to and perception of stimuli

MOA

GLUCOCORTICOIDS •  sensitivity of the Y-aminobutyric acid GABAA


Drugs Hydrocortisone (A-hydrocort ®) receptors to the inhibitory neurotransmitters GABA
Budenoside (pulmicort ®) • Unlike other anaesthetics, nitrous oxide and ketamine
have effects that are likely mediated via inhibition of
the N-methyl0D-aspartate (NMDA) receptors
PURINE ANALOGS
• Activity of the inhibitory glycine receptors in spinal
Drugs Azathioprine (Imuran ®)
6-Mercaptopurine (purinethol®) motor neurons is 

5 Primary Effects
ANESTHETICS 1. Unconsciousness
2. Amnesia
3. Analgesia
4. Skeletal muscle relaxation
5. Inhibition of reflexes

Status of organ systems

• Cardiovascular system
• Respiratory system
• Liver and kidney
• Nervous system
Prepared by: MJCN
UIC PharmChem CARES
GREEN REVIEW 2021: Pharmacology I 41

• Pregnancy Nitrous oxide 0.47 fast


Halothane 2.3 slow
Stages

1. Analgesia Halothane 2.4; most soluble in blood


•  awareness of pain Isoflurane 1.4
2. Excitement (delirium) Sevoflurane 0.65
• Combative Nitrous oxide 0.47
• Rapid respiration desflurane 0.42
•  HR and BP
3. Surgical anesthesia
• Regular respiration and controlled HR INHALATION ANESTHESIA
• Ideal stage for surgery → Act in different ways at the level of the central
• Careful monitoring is needed to prevent nervous system
undesired progression to stage 4 → May disrupt normal synaptic transmission by:
4. Medullary paralysis/ depression o Interfering with the release of
• Severe depression of the respiratory center neurotransmitters from pre-synaptic nerve
• Ventilation and/ circulation must be terminal (enhance or depress excitatory or
supported to prevent death inhibitory transmission)
Pharmacokinetics o Altering the reuptake of neurotransmitters
o Changing the binding of neurotransmitters to
◆ Uptake into the blood the post synaptic receptor sites
Inhalational anesthetics: taken up passively via o Influencing the ionic conductance change
diffusion depending on: that follows activation of the post synaptic
1. Blood solubility of the anesthetic receptor by neurotransmitters
2. Blood-gas partition coefficient Both pre and post synaptic effects have been
3. Lung ventilation, volumes, and perfusion found
◆ Distribution and uptake into the brain
→ Transport to and uptake into the brain Pharmacokinetics:
depend on cerebral perfusion and the fat • Depth of anesthesia depend on MAC and partial
solubility of the inhalational anesthetic pressure of the gas in brain
→ Brain-blood partition coefficient • Passes through series of tension gradients
◆ Onset of action/ effect
→ The lower the blood-gas partition coefficient Minimum alveolar concentration
of an inhalational anesthetic, the faster the
→ Concentration of inhaled anesthetic needed to
substance takes effect (less induction time
eliminate movement in 50% of patients stimulated by
◆ Elimination
standardized incision (painful stimuli)
→ Inhalational anesthetics are eliminated by the
→ The median effective dose (ED50) of an anesthetic,
lungs
expresses as the percentage of gas in mixture
→ Lower blood-gas partition coefficient of an
required to achieve that effect
inhalational anesthetic, the faster the effect
→ Numerically, MAC is small for potent anesthetics such
ceases (less recovery time)
as sevoflurane and large for less potent agents such
→ Inhalational anesthetics are metabolized only
as nitrous oxide
to small degree exception: halothane is
metabolized in liver OLDER GENERATION NEW GENERATION
◆ Blood: gas partition coefficient Halothane Isoflurane
→ Solubility of an ansethetic agent in blood is quantified Methoxyflurane Desflurane
as blood: gas partition coefficient Enflurane sevoflurane
→ It is the ratio of the concentration of an anesthetic in
the blood phase to the concentration of the anesthetic
Halothane
in the gas phase when the anesthetic is in equilibrium
→ Potent bronchodilator
between the two phases
→ Relaxes both skeletal and uterine muscles and can be
blood: gas coefficient; faster induction and recovery
used in obstetrics when uterine relaxation is indicated
(nitrous oxide)
→ Has pleasant odor, it is suitable in paediatrics for
ihalation induction although sevoflurane is now the
blood: gas coefficient; slower induction and
DOC
recovery (halothane)
→ Hepatic necrosis (adults)
→ Hypotension
Alveolus Blood brain → Arrhythmia (exaggerated catcholamine)
Blood: gas Rate of induction → Malignant hyperthermia
partition
coefficient Enflurane
Prepared by: MJCN
UIC PharmChem CARES
GREEN REVIEW 2021: Pharmacology I 42

→ Lesser risk of arrhythmia → Widely used and has replaced thiopental as the DOC
→ Causes seizures for induction of general anesthesia and sedation
→ Occasionally accompanied by excitatory phenomena
such as muscle twitching, spontaneous movement,
Isoflurane yawning and hiccups
→ Preferred anesthetic in neurosurgery (does not raise
intracranial pressure)
LOCAL ANESTHETICS
sevoflurane
→ Preferred for children (low pungency or pleasant → Loss of sensation in body part without the loss of
odor, allowing rappid induction without irritating consciousness or the impairment of central control of
airways) vital functions
◆ Administration: topical or epidural (injected in
Nitrous oxide nerves), has vasodilating properties, except cocaine
→ Laughing gas
→ Dental surgery Cocaine
→ s/e:
→ Inhibits the reuptake of NE→ vasoconstriction
o post operation N//V
→ Vasodilation leads to rapid diffusion away from the
o oxidizes cobalt in B12- megaloblastic anemia
site of action and shorter duration when these drugs
o fetal abnormalities- aplastic anemia
are administered alone
o diffusion hypoxia- replaces O2 in lungs
→ By adding vasoconstrictor epinephrine, the rate of
INTRAVENOUS ANEESTHESIA local anesthetic absorption and diffusion is 
- This minimizes systemic toxicity
Thiopental and  duration of action
→ ultra short acting barbiturate
◆ Lipid solubilty
→ induction of anesthesia
→ All local anesthetics have weak bases
→  lipid solubility = faster penetration of
nerve, blocks Na channels, speed up onset
Midazolam of action
→ cause anterograde amnesia ◆ Influence of Ph
→ patient retains memory of past events but new info is → pKa (7.6-7.8)- faster acting: lidocaine,
transferred into long term memory mepivacaine
→  pKa (8.1- 8.9) – slower acting procaine,
tetracaine, bupivacaine
Etomidate ◆ Vasoconstrictors
→ potent ultra-short acting non-barbiturate → Cocaine itself is vasoconstrictor
→ GABA-mimetic → Adrenaline: potential a/e of vasoconstrictors
→ Poor H2O solubility → Don’t use in areas of toes, fingers , ear
lobes, ischemia (penis) and necrosis
Opioid ◆ Inflammation
→ Fentanyl and morphine
→ Tends to produce  pH in tissues
Ketamine → More ionized- don’t penetrate well
→ Dissociative anesthesia → Blood flow
→ Unconscious but may appear to be awake and does →  ability to produce fx
not feel pain
MOA:
→ NMDA receptor inhibitor (also nitrous oxide)
→ May be used illicitly, since it causes a dream like state • Blocks nerve conduction of sensort impulses
and hallucinations similar to phencyclidine PCP • Na+ ion channels are directly blocked to prevent the
transient increase in permeability of the nerve
Dexmetomidine
membrane to Na+ that is required for an action
→ Used in sensitive care settings and surgery
potential
Propofol • When propagation of action potentials is prevented,
→ IV sedative/ hypnotic used for the induction and/ sensation cannot be transmitted from the source if
maintenance of anesthesia stimulation to the brain
→ Rapid onset, rapid recovery
Uses:
→ Profound respiratory depression w/ apnea occurring
for 30s • Excision
→ Poorly water-soluble, suppied as an emulsion • Dentistry
containing soybean oil and egg phospholipid giving it • Dermatology
a milk-like appearance • Spinal anesthesia

Prepared by: MJCN


UIC PharmChem CARES
GREEN REVIEW 2021: Pharmacology I 43

General local Lidocaine


Site of action CNS Peripheral nerves → Most widely used local anesthetic
Area Whole body Restricted areas → Useful for individuals sensitive to ester-type
Consciousness Lost Unaltered → Used as anti-arrhythmic
Preferential use Major surgery Minor surgery → Metabolites: xylides- still retain local anesthetic
activity
Use in non Possible Not possible → Toxicity: drowsiness, tinnitus, dizziness
cooperative px
→ With increased dose: seizures,coma, respiratory
Poor health px Risky Safe
depression
Care for vital Essential Not needed
functions Prilocaine
→ Similar profile with lidocaine but has lesser
vasodilatory effect (can be used without
Esters Amides
vasoconstrictor) and increased volume of distribution
Metabolism Fast, hydrolysis of Slow, Liver
blood esterases enzymes reducing its CNS toxicity
Half life Short Long → Metabolite: O-toluidine: causes methemoglobinemia
Cocaine Lidocaine (tx: methylene blue)
Procaine (xylocaine ®) → Dental anesthetic
Benzocaine Bupivacaine
EMLA (Eutectic mixture of local anesthetic)
Dibucaine (sensorcaine ®)
Tetracaine Mepivacaine → Eutectic: 2 solid substances mixed together in equal
chlorprocaine Prilocaine quantities by wt.
Etidocaine
→ Melting point is higher than the individual melting
Articaine
points
Ropivacaine
(naropin®) → Lidocaine + prilocaine= oily mixture
Systemic toxicity Less likely More likely
Allergic rxn Possible PABA Very rare
derivative
Stability in soln. Breaks down in Very stable
ampules, heat
Onset Slow at general Mod-fast
rule
pKa 8.5-8.9 Close to
physiologic
Higher than 7.6-8.1
physiologic pH

ESTERS

→ Benzoic acid derivatives; cocaine


→ Para-amino benzoic acid derivatives: procaine

procaine
→ First synthetic local anesthetic
→ Hydrolysed to PABA w/c inhibits the action of
sulfonamides

cocaine
→ Cause vasoconstriction
→ Not very useful at present because of abuse potential
and toxicity

AMIDES

Bupivacaine
→ Noted for cardiotoxicity (arrhythmia hypotension) if
accidentally injected IV
→ Can be given as continuous infusion for providing
prolonged anesthesia during labor or the post-
operative period

mepivacaine
→ Intermediate-acting
→ Not used in obstetrics, toxic neonates
Prepared by: MJCN
UIC PharmChem CARES
GREEN REVIEW 2021: Pharmacology I 44

QUESTIONNAIRES neurons release NE and parasympathetic neurons release ACh


in the effector cells. Afferent neurons carry signals from the
1. This receptor family is also known as metabotropic
periphery to the CNS.
because it either increase or decrease the
concentration of metabolites 5. Which of the following is correct regarding somatic
a. TYPE IV motor neurons?
b. TYPE III a. The neurotransmitter at the somatic motor
c. TYPE II neuron ganglion is acetylcholine.
d. TYPE I b. The neurotransmitter at the somatic motor
neuron ganglion is norepinephrine.
Answer: c. TYPE II; G- protein coupled receptor is also known c. Somatic motor neurons innervate smooth
as metabotropic and is under type 2 receptor
muscles.
2. Isoproterenol produces maximal contraction of d. Somatic motor neurons do not have ganglia.
cardiac muscle in a manner similar to epinephrine. e. Responses in the somatic motor neurons are
Which of the following best describes isoproterenol? generally slower than in the autonomic
a. Full agonist. nervous system.
b. Partial agonist. Answer: d. Somatic motor neurons do not have ganglia;
c. Competitive antagonist. Somatic motor neurons innervate skeletal muscles (not smooth
d. Irreversible antagonist.
muscle) and have no ganglia. Answers A and B are incorrect,
e. Inverse agonist. since there are no ganglia. Also, the responses in the somatic
Answer: a. Full agonist; full agonist has an Emax similar to the motor nervous system are faster compared to the responses in
endogenous ligand. A partial agonist would only produce a the autonomic nervous system due to the lack of ganglia in the
partial effect. An antagonist would block the effects of an former.
endogenous agonist. An inverse agonist would reverse the
constitutive activity of receptors and exert the opposite
pharmacological effect. 6. Which of the following changes could theoretically
happen in a person when the parasympathetic system
is inhibited using a pharmacological agent?
3. If 10 mg of naproxen produces the same analgesic a. Reduction in heart rate.
response as 100 mg of ibuprofen, which of the b. Constriction of the pupil (miosis).
following statements is correct? c. Increase in gastric motility.
a. Naproxen is more efficacious than is d. Dry mouth (xerostomia).
ibuprofen. e. Contraction of detrusor muscle in the bladder.
b. Naproxen is more potent than ibuprofen. Answer: d. Dry mouth (xerostomia); Activation of the
c. Naproxen is a full agonist, and ibuprofen is a parasympathetic system causes a reduction in heart rate,
partial agonist.
constriction of the pupil, an increase in gastric motility and
d. Naproxen is a competitive antagonist. salivation, and contraction of the bladder muscle. Therefore,
e. Naproxen is a better drug to take for pain relief inhibition of the parasympathetic system causes an increase in
than is ibuprofen. heart rate, dilation of the pupil, a decrease in gastric motility, dry
Answer: b.Naproxen is more potent than ibuprofen; Without mouth, and relaxation of detrusor muscles.
information about the maximal effect of these drugs, no 7. Which of the following statements is correct regarding
conclusions can be made about efficacy or intrinsic activity. E is the sympathetic and parasympathetic systems?
false because the maximal response obtained is often more
a. Acetylcholine activates muscarinic receptors.
important than the amount of drug needed to achieve it. b. Acetylcholine activates adrenergic receptors.
c. Norepinephrine activates muscarinic
receptors.
4. Which of the following is correct regarding the d. Activation of the sympathetic system causes
autonomic nervous system (ANS)? a drop in blood pressure.
a. Afferent neurons carry signals from the CNS to the
effector organs. Answer: a. Acetylcholine activates muscarinic receptors;
b. The neurotransmitter at the parasympathetic Acetylcholine is the neurotransmitter in the cholinergic system,
ganglion is norepinephrine (NE). and it activates both muscarinic and nicotinic cholinergic
c. The neurotransmitter at the sympathetic ganglion is receptors, not adrenergic receptors. Norepinephrine activates
acetylcholine (ACh). adrenergic receptors, not muscarinic receptors. Activation of the
d. Sympathetic neurons release ACh in the effector sympathetic system causes an increase in blood pressure (not
organs. a drop in blood pressure) due to vasoconstriction and
e. Parasympathetic neurons release NE in the effector stimulation of the heart.
organs. 8. An elderly man was brought to the emergency room
Answer: c. The neurotransmitter at the sympathetic and after he ingested a large quantity of carvedilol tablets,
parasympathetic ganglia is acetylcholine; Sympathetic a drug that blocks α1, β1, and β2 adrenergic receptors,

Prepared by: MJCN


UIC PharmChem CARES
GREEN REVIEW 2021: Pharmacology I 45

which mainly mediate the cardiovascular effects of 11. If an ophthalmologist wants to dilate the pupils for an
epinephrine and norepinephrine in the body. Which of eye examination, which of the following drugs/classes
the following symptoms would you expect in this of drugs could be theoretically useful?
patient? a. Muscarinic receptor activator (agonist).
a. Increased heart rate (tachycardia). b. Muscarinic receptor inhibitor (antagonist).
b. Reduced heart rate (bradycardia). c. Acetylcholine.
c. Dilation of the pupil (mydriasis). d. Pilocarpine.
d. Increased blood pressure. e. Neostigmine.

Answer: b. Reduced heart rate (bradycardia); Activation of α1 Answer: b. Muscarinic receptor inhibitor (antagonist);
receptors causes mydriasis, vasoconstriction, and an increase Muscarinic agonists (for example, ACh, pilocarpine) contract the
in blood pressure. Activation of β1 receptors increases heart circular smooth muscles in the iris sphincter and constrict the
rate, contractility of the heart, and blood pressure. Activation of pupil (miosis). Anticholinesterases (for example, neostigmine,
β2 receptors causes dilation of bronchioles and relaxation of physostigmine) also cause miosis by increasing the level of ACh.
skeletal muscle vessels. Thus, inhibition of these receptors will Muscarinic antagonists, on the other hand, relax the circular
cause vasorelaxation (α1 blockade), reduction in heart rate (β1 smooth muscles in the iris sphincter and cause dilation of the
blockade), reduction in contractility of the heart (β1 blockade), pupil (mydriasis).
reduction in blood pressure, bronchoconstriction (β2 blockade),
12. In Alzheimer’s disease, there is a deficiency of
and constriction of blood vessels supplying skeletal muscles (β2
cholinergic neuronal function in the brain.
blockade).
Theoretically, which of the following strategies will be
9. Botulinum toxin blocks the release of acetylcholine useful in treating the symptoms of Alzheimer’s
from cholinergic nerve terminals. Which of the disease?
following is a possible effect of botulinum toxin? a. Inhibiting cholinergic receptors in the brain.
a. Skeletal muscle paralysis. b. Inhibiting the release of acetylcholine in the
b. Improvement of myasthenia gravis symptoms. brain.
c. Increased salivation. c. Inhibiting the acetylcholinesterase enzyme in
d. Reduced heart rate the brain.
d. Activating the acetylcholinesterase enzyme in
Answer: a. Skeletal muscle paralysis; Acetylcholine released by the brain.
cholinergic neurons acts on nicotinic receptors in the skeletal
muscle cells to cause contraction. Therefore, blockade of ACh Answer: c. Inhibiting the acetylcholinesterase enzyme in the
release causes skeletal muscle paralysis. Myasthenia gravis is brain. Since there is already a deficiency in brain cholinergic
an autoimmune disease where antibodies are produced against function in Alzheimer’s disease, inhibiting cholinergic receptors
nicotinic receptors and inactivate nicotinic receptors. A or inhibiting the release of ACh will worsen the condition.
reduction in ACh release therefore worsens (not improves) the Activating the acetylcholinesterase enzyme will increase the
symptoms of this condition. Reduction in ACh release by degradation of ACh, which will again worsen the condition.
botulinum toxin causes reduction in secretions including saliva However, inhibiting the acetylcholinesterase enzyme will help to
(not increase in salivation) causing dry mouth and an increase increase the levels of ACh in the brain and thereby help to relieve
(not reduction) in heart rate due to reduced vagal activity. the symptoms of Alzheimer’s disease.

10. Which of the following is a systemic effect of a


muscarinic agonist?
a. Reduced heart rate (bradycardia). 13. An elderly female who lives in a farm house was
brought to the emergency room in serious condition
b. Increased blood pressure.
after ingesting a liquid from an unlabeled bottle found
c. Mydriasis (dilation of the pupil).
near her bed, apparently in a suicide attempt. She
d. Reduced urinary frequency.
presented with diarrhea, frequent urination,
e. Constipation
convulsions, breathing difficulties, constricted pupils
Answer: a. Reduced heart rate (bradycardia); A muscarinic (miosis), and excessive salivation. Which of the
agonist binds to and activates muscarinic receptors in the heart, following is correct regarding this patient?
endothelial cells (blood vessels), the gut, and iris sphincter (eye) a. She most likely consumed an
and urinary bladder wall muscles, in addition to several other organophosphate pesticide.
tissues. Activation of muscarinic receptors by an agonist causes b. The symptoms are consistent with
a reduction in heart rate, constriction of circular muscles in the sympathetic activation.
iris sphincter leading to constriction of the pupil (miosis), c. Her symptoms can be treated using an
increased GI motility (hence, diarrhea, not constipation), and anticholinesterase agent.
contraction of bladder muscles leading to an increase (not d. Her symptoms can be treated using a
decrease) in urination frequency. In the endothelial cells of cholinergic agonist.
blood vessels, muscarinic activation produces release of nitric
oxide that causes vasorelaxation and a reduction (not increase) Answer: a. She most likely consumed an organophosphate
pesticide; The symptoms are consistent with that of cholinergic
in blood pressure.
crisis. Since the elderly female lives on a farm and since the
symptoms are consistent with that of cholinergic crisis (usually

Prepared by: MJCN


UIC PharmChem CARES
GREEN REVIEW 2021: Pharmacology I 46

caused by cholinesterase inhibitors), it may be assumed that d. Phentolamine.


she has consumed an organophosphate pesticide (irreversible e. Bethanechol.
cholinesterase inhibitor). Assuming that the symptoms are
Answer: c. Tropicamide; Muscarinic agonists such as ACh,
caused by organophosphate poisoning, administering an
pilocarpine, and bethanechol contract the circular muscles of iris
anticholinesterase agent or a cholinergic agonist will worsen the
sphincter and cause constriction of the pupil (miosis), whereas
condition. The symptoms are not consistent with that of
muscarinic antagonists such as atropine and tropicamide
sympathetic activation, as sympathetic activation will cause
prevent the contraction of the circular muscles of the iris and
symptoms opposite to that of cholinergic crisis seen in this
cause dilation of the pupil (mydriasis). α-Adrenergic antagonists
patient.
such as phentolamine relax the radial muscles of the iris and
cause miosis.

14. Which of the following drugs or classes of drugs will 17. Sarin is a nerve gas that is an organophosphate
be useful in treating the symptoms of myasthenia cholinesterase inhibitor. Which of the following could
gravis? be used as an antidote to sarin poisoning?
a. Nicotinic antagonists. a. Pilocarpine.
b. Muscarinic agonists. b. Carbachol.
c. Muscarinic antagonists. c. Atropine.
d. Anticholinesterase agents. d. Physostigmine.
e. Nicotine.
Answer: d. Anticholinesterase agents. The function of nicotinic
receptors in skeletal muscles is diminished in myasthenia gravis Answer: c. Atropine; Sarin is an organophosphate
due to the development of antibodies to nicotinic receptors in cholinesterase inhibitor. It causes an increase in ACh levels in
the patient’s body (autoimmune disease). Any drug that can tissues that leads to cholinergic crisis by the activation of
increase the levels of ACh in the neuromuscular junction can muscarinic as well as nicotinic receptors. Most of the symptoms
improve symptoms in myasthenia gravis. Thus, cholinesterase of cholinergic crisis are mediated by muscarinic receptors and,
inhibitors help to improve the symptoms of myasthenia gravis. therefore, the muscarinic antagonist atropine is used as an
Muscarinic drugs have no role in myasthenia gravis, and antidote for sarin poisoning. Cholinergic agonists such as
nicotinic antagonists will worsen the symptoms. pilocarpine, carbachol, physostigmine (indirect agonists), and
nicotine will worsen the symptoms of sarin poisoning.

15. Atropa belladonna is a plant that contains atropine (a


muscarinic antagonist). Which of the following drugs 18. A patient with chronic obstructive pulmonary disease
or classes of drugs will be useful in treating poisoning (COPD) was prescribed a β2 agonist for the relief of
with belladonna? bronchospasm. However, the patient did not respond
a. Malathion. to this treatment. Which of the following drugs or
b. Physostigmine. classes of drugs would you suggest for this patient as
c. Muscarinic antagonists. the next option?
d. Nicotinic antagonists a. β1 Agonist.
b. Muscarinic agonist.
Answer: b. Physostigmine; Atropine is a competitive muscarinic c. Physostigmine.
receptor antagonist that causes anticholinergic effects.
d. Ipratropium.
Muscarinic agonists or any other drugs that can increase the
e. Phentolamine.
levels of ACh will be able to counteract the effects of atropine.
Thus, anticholinesterases such as malathion and physostigmine Answer: d. Ipratropium; Major receptors present in the
can counteract the effects of atropine in theory. However, bronchial tissues are muscarinic and adrenergic-β2 receptors.
malathion being an irreversible inhibitor of acetylcholinesterase Muscarinic activation causes bronchoconstriction, and β2
is not used for systemic treatment in patients. Muscarinic receptor activation causes bronchodilation. Therefore, direct or
antagonists will worsen the toxicity of atropine. Nicotinic indirect (physostigmine) muscarinic agonists will worsen
antagonists could worsen the toxicity by acting on bronchospasm. Ipratropium is a muscarinic antagonist that can
parasympathetic ganglionic receptors and thus reducing the relax bronchial smooth muscles and relieve bronchospasm in
release of ACh. patients who are not responsive to β2 agonists. α1 and β1
receptors are not commonly present in bronchial tissues and,
therefore, β1 agonists or α antagonists (phentolamine) do not
16. During an ophthalmic surgical procedure, the have any significant effects on bronchospasm
surgeon wanted to constrict the pupil of the patient 19. Which of the following drugs would be the most
using a miotic drug. However, he accidentally used effective anti–motion sickness drug for a person
another drug that caused dilation of the pupil
planning to go on a cruise?
(mydriasis) instead. Most likely, which of the following a. Atropine.
drugs did he use? b. Tropicamide.
a. Acetylcholine. c. Scopolamine.
b. Pilocarpine.
d. Darifenacin.
c. Tropicamide. e. Tiotropium.
Prepared by: MJCN
UIC PharmChem CARES
GREEN REVIEW 2021: Pharmacology I 47

Answer: c. Scopolamine; All muscarinic antagonists 22. Which of the following is correct regarding the
(anticholinergic drugs) listed above are theoretically useful as neuromuscular blockers (NMBs)?
anti–motion sickness drugs; however, scopolamine is the most a. Nondepolarizing NMBs are administered
effective in preventing motion sickness in practice. Tropicamide orally.
mostly has ophthalmic uses, and tiotropium is used for b. Cholinesterase inhibitors reduce the effects
respiratory disorders (COPD). Darifenacin is used for overactive of nondepolarizing NMBs.
bladder. c. Nondepolarizing NMBs affect diaphragm
muscles first.
d. Effects of depolarizing neuromuscular
20. A patient was administered a neuromuscular blocker blockers can be reversed using
(NMB) prior to a surgical procedure to produce cholinesterase inhibitors.
skeletal muscle paralysis. This NMB drug affected Answer: b. Cholinesterase inhibitors reduce the effects of
small, rapidly contracting muscles of the face and nondepolarizing NMBs. Nondepolarizing NMBs such as
eyes first and diaphragm muscles last. The effect of cisatracurium and vecuronium are highly polar compounds and
this drug was easily reversed with neostigmine. Which
are poorly absorbed from the GI tract. Therefore, they are
of the following neuromuscular blockers was most
administered parenterally, not orally. Nondepolarizing NMBs
likely administered to this patient? are competitive antagonists at nicotinic receptors. Therefore,
a. Rocuronium.
increasing the levels of ACh at the neuromuscular junction
b. Succinylcholine. reduces the effects of these agents. Cholinesterase inhibitors
c. Diazepam. increase the levels of ACh at the neuromuscular junction and
d. Tubocurarine. reduce the effects of nondepolarizing NMBs, but may enhance
Answer: a. Rocuronium.. There are two types of NMBs: (not reverse) the effects of depolarizing NMBs.
depolarizing and nondepolarizing NMBs. Depolarizing NMBs are Nondepolarizing NMBs first affect rapidly contracting muscles
agonists at the nicotinic receptors, whereas nondepolarizing seen in the face and eyes and affect the diaphragm muscles
NMBs are antagonists at the nicotinic receptors. Both types of last.
NMBs affect the rapidly contracting muscles (face, eye, etc.)
first and diaphragm muscles last. However, cholinesterase
inhibitors such as neostigmine increase ACh levels in the NMJ 23. Which of the following adrenergic agonists is most
and reverse the effects of nondepolarizing NMBs, but not those likely to cause CNS side effects when administered
of depolarizing NMBs. Therefore, the NMB administered to this systemically?
patient is most probably rocuronium, which is a nondepolarizing a. Epinephrine.
NMB. Tubocurarine is also a nondepolarizing NMB, but it is not b. Norepinephrine.
used in practice. Succinylcholine is a depolarizing NMB, and c. Isoproterenol.
diazepam is a benzodiazepine that does not cause paralysis of d. Dopamine.
skeletal muscles. e. Ephedrine.

Answer: e. Ephedrine. Ephedrine is more lipophilic compared


to the other drugs listed and therefore is more likely to cross
21. A patient was administered a neuromuscular blocker the blood–brain barrier when administered systemically.
(NMB) prior to a surgical procedure to produce
Therefore, ephedrine is more likely to cause CNS side effects
skeletal muscle paralysis. This NMB drug caused compared to other listed drugs.
initial skeletal muscle fasciculations before the onset
of paralysis. The effect of this drug could not be 24. A 12-year-old boy who is allergic to peanuts was
reversed with neostigmine. Which of the following brought to the emergency room after accidentally
neuromuscular blockers was most likely administered consuming peanuts contained in fast food. He is in
to this patient? anaphylactic shock. Which of the following drugs
a. Cisatracurium. would be most appropriate to treat this patient?
b. Succinylcholine. a. Norepinephrine.
c. Diazepam. b. Phenylephrine.
d. Tubocurarine. c. Dobutamine.
d. Epinephrine.
Answer: b. Succinylcholine. Depolarizing NMBs cause muscle
fasciculations before causing paralysis, and their effects cannot Answer: d. Epinephrine. Norepinephrine has more α agonistic
be reversed using cholinesterase inhibitors such as neostigmine. effects and activates mainly α1, α2, and β1 receptors.
Nondepolarizing NMBs do not cause muscle fasciculations, and Epinephrine has more β agonistic effects and activates mainly
their effects can be reversed using cholinesterase inhibitors. α1, α2, β1, and β2 receptors. Phenylephrine has predominantly
Therefore, the NMB used in this patient is succinylcholine, which α effects and activates mainly α1 receptors. Dobutamine
is a depolarizing NMB. Cisatracurium and tubocurarine are mainly activates β1 receptors and has no significant effects on
nondepolarizing NMBs, and diazepam does not cause paralysis β2 receptors. Thus, epinephrine is the drug of choice in
of skeletal muscles anaphylactic shock that can both stimulate the heart (β1
activation) and dilate bronchioles (β2 activation).

Prepared by: MJCN


UIC PharmChem CARES
GREEN REVIEW 2021: Pharmacology I 48

25. A 70-year-old patient was brought to the emergenc not have any stimulatory effects on β2 receptors. Also,
room with a blood pressure of 76/60 mm Hg, norepinephrine is not active when given orally.
tachycardia, and low cardiac output. He was
diagnosed with acute heart failure. Which of the
following drugs would be the most appropriate to 28. A new antihypertensive drug was tested in an animal
improve his cardiac function? model of hypertension. The drug when given alone
a. Epinephrine. reduces blood pressure in the animal.
b. Fenoldopam. Norepinephrine when given in the presence of this
c. Dobutamine. drug did not cause any significant change in blood
d. Isoproterenol. pressure or heart rate in the animal. The new drug is
similar to which of the following drugs in terms of its
Answer: c. Dobutamine. Among the choices, the ideal drug to
pharmacological mechanism of action?
increase contractility of the heart in acute heart failure is
a. Prazosin.
dobutamine, since it is a selective β1-adrenergic agonist.
b. Clonidine.
Fenoldopam is a dopamine agonist used to treat severe
c. Propranolol.
hypertension. Other drugs are nonselective adrenergic
d. Metoprolol.
agonists that could cause unwanted side effects
e. Carvedilol.

Answer: e. Carvedilol. Norepinephrine activates both α1 and


26. Which of the following adrenergic agonists is β1 receptors and causes an increase in heart rate and blood
commonly present in nasal sprays available over-the- pressure. A drug that prevents the increase in blood pressure
counter (OTC) to treat nasal congestion? caused by norepinephrine should be similar to carvedilol that
a. Clonidine. antagonizes both α1 and β1 receptors. Prazosin is an α1
b. Albuterol. antagonist, clonidine is an α2 agonist, and propranolol and
c. Oxymetazoline. metoprolol are β antagonists, and these drugs cannot
d. Dobutamine. completely prevent the cardiovascular effects of
e. Norepinephrine norepinephrine.

Answer: c. Oxymetazoline.. Drugs with selective α1 agonistic


activity are commonly used as nasal decongestants because of
29. A β-blocker was prescribed for hypertension in a
their ability to cause vasoconstriction in the nasal vessels.
female asthma patient. After about a week of
Oxymetazoline is an α1 agonist and therefore the preferred
treatment, the asthma attacks got worse, and the
drug among the choices as a nasal decongestant. Clonidine is
patient was asked to stop taking the β-blocker.
an α2 agonist, albuterol is a β2 agonist, dobutamine is a β1
agonist, and norepinephrine is a nonselective adrenergic Which of the following β-blockers would you suggest
as an alternative in this patient that is less likely to
agonist.
worsen her asthma?
27. One of your patients who is hypertensive and gets a. Propranolol.
mild asthma attacks occasionally bought an herbal b. Metoprolol.
remedy online to help with his asthma. He is not on c. Labetalol.
any asthma medications currently but is receiving a d. Carvedilol.
β1-selective blocker for his hypertension. The herbal
Answer: b. Metoprolol.. The patient was most likely given a
remedy seems to relieve his asthma attacks, but his
nonselective β-blocker (antagonizes both β1 and β2 receptors)
blood pressure seems to increase despite the β-
that made her asthma worse due to β2 antagonism. An
blocker therapy. Which of the following drugs is
alternative is to prescribe a cardioselective (antagonizes only
most likely present in the herbal remedy he is
β1) β-blocker that does not antagonize β2 receptors in the
taking?
bronchioles. Metoprolol is a cardioselective β-blocker.
a. Phenylephrine.
Propranolol, labetalol, and carvedilol are nonselective β-
b. Norepinephrine.
blockers and could worsen the asthma.
c. Dobutamine.
d. Ephedrine.
e. Salmeterol.
30. A 70-year-old male needs to be treated with an α-
Answer: d. Ephedrine; Two drugs among the choices that blocker for overflow incontinence due to his enlarged
could relieve asthma are ephedrine and salmeterol, as they prostate. Which of the following drugs would you
activate β2 receptors in the bronchioles and cause suggest in this patent that will not affect his blood
bronchodilation. However, salmeterol is a selective β2 agonist pressure significantly?
and should not cause an increase in blood pressure. Ephedrine a. Prazosin.
on the other hand stimulates the release of norepinephrine and b. Doxazosin.
acts as a direct agonist at α- and β-adrenergic receptors, thus c. Phentolamine.
causing an increase in blood pressure. Phenylephrine (a d. Tamsulosin.
nonselective α agonist) does not cause bronchodilation. e. Terazosin.
Norepinephrine is a nonselective adrenergic agonist that does

Prepared by: MJCN


UIC PharmChem CARES
GREEN REVIEW 2021: Pharmacology I 49

Answer: d. Tamsulosin; Tamsulosin is an α1 antagonist that is Ropinirole directly stimulates dopamine receptors, but it does
more selective to the α1 receptor subtype (α1A) present in the not cause vasospasm. The other drugs do not act directly on
prostate and less selective to the α1 receptor subtype (α1B) dopamine receptors.
present in the blood vessels. Therefore, tamsulosin does not
affect blood pressure significantly. Prazosin, doxazosin,
terazosin, and phentolamine antagonize both these subtypes 34. Modest improvement in the memory of patients with
and cause significant hypotension as a side effect. Alzheimer’s disease may occur with drugs that
increase transmission at which of the following
receptors?
31. Which of the following drugs is commonly used a. Adrenergic.
topically in the treatment of glaucoma? b. Cholinergic.
a. Atropine. c. Dopaminergic.
b. Timolol. d. GABAergic.
c. Tropicamide. e. Serotonergic.
d. Scopolamine.

Answer: b. Timolol.. β-Blockers reduce the formation of


aqueous humor in the eye and therefore reduce intraocular Answer: b. Cholinergic; AChE inhibitors, such as rivastigmine,
increase cholinergic transmission in the CNS and may cause a
pressure, thus relieving glaucoma. Timolol is a nonselective β-
modest delay in the progression of Alzheimer’s disease.
blocker that is commonly used topically to treat glaucoma.
Increased transmission at the other types of receptors listed
Atropine, tropicamide, and scopolamine are anticholinergic
does not result in improved memory.
drugs that might worsen glaucoma.

35. Which medication is a glutamate receptor antagonist


32. A 50-year-old male was brought to the emergency
that can be used in combination with an
room after being stung by a hornet. The patient was
acetylcholinesterase inhibitor to manage the
found to be in anaphylactic shock, and the medical
symptoms of Alzheimer’s disease?
team tried to reverse the bronchoconstriction and
a. Rivastigmine.
hypotension using epinephrine. However, the patient
b. Ropinirole.
did not fully respond to the epinephrine treatment.
c. Fluoxetine.
The patient’s wife mentioned that he is taking a
d. Memantine.
prescription medication for his blood pressure, the
e. Donepezil.
name of which she does not remember. Which of the
following medications is he most likely taking that Answer: d. Memantine. When combined with an
could have prevented the effects of epinephrine? acetylcholinesterase inhibitor, memantine has modest efficacy
a. Doxazosin. in keeping patients with Alzheimer’s disease at or above
b. Propranolol. baseline for at least 6 months and may delay disease
c. Metoprolol. progression.
d. Acebutolol.

Answer: b. Propranolol. Epinephrine reverses hypotension by


activating β1 receptors and relieves bronchoconstriction by 36. Which of the following agents is available as a patch
activating β2 receptors in anaphylaxis. Since epinephrine was for once-daily use and is likely to provide steady
not effective in reversing hypotension or bronchoconstriction drug levels to treat Alzheimer’s disease?
in this patient, it could be assumed that the patient was on a a. Rivastigmine.
nonselective β-blocker (propranolol). Doxazosin (α1-blocker), b. Donepezil.
metoprolol, or acebutolol (both β1-selective blockers) would c. Memantine.
not have completely prevented the effects of epinephrine. d. Galantamine.
e. Glatiramer.

Answer: a. Rivastigmine. Rivastigmine is the only agent


33. Which of the following antiparkinsonian drugs may available as a transdermal delivery system for the treatment of
cause vasospasm? Alzheimer’s disease. It may also be used for dementia
a. Amantadine. associated with Parkinson’s disease
b. Bromocriptine.
c. Carbidopa.
d. Entacapone.
37. Which one of the following statements is correct
e. Ropinirole.
regarding benzodiazepines?
a. Benzodiazepines directly open chloride
channels.
Answer: b. Bromocriptine. Bromocriptine is a dopamine b. Benzodiazepines show analgesic actions.
receptor agonist that may cause vasospasm. It is c. Clinical improvement of anxiety requires 2 to
contraindicated in patients with peripheral vascular disease. 4 weeks of treatment with benzodiazepines.
Prepared by: MJCN
UIC PharmChem CARES
GREEN REVIEW 2021: Pharmacology I 50

d. All benzodiazepines have some sedative psychiatric symptoms, the patient complained of
effects. muscle aches throughout his body. Physical and
e. Benzodiazepines, like other CNS laboratory tests were unremarkable. After 6 weeks of
depressants, readily produce general therapy with fluoxetine, his symptoms resolved.
anesthesia. However, the patient complains of sexual
dysfunction. Which of the following drugs might be
Answer: d. All benzodiazepines have some sedative effects.
useful in this patient?
Although all benzodiazepines can cause sedation, the drugs a. Fluvoxamine.
labeled “benzodiazepines” are promoted for the treatment of b. Sertraline.
sleep disorder. Benzodiazepines enhance the binding of c. Citalopram.
GABAA to its receptor, which increases the permeability of
d. Mirtazapine.
chloride. The benzodiazepines do not relieve pain but may e. Lithium.
reduce the anxiety associated with pain. Unlike the tricyclic
antidepressants and the monoamine oxidase inhibitors, the
benzodiazepines are effective within hours of administration.
Answer: d. Mirtazapine. Mirtazapine is largely free from sexual
Benzodiazepines do not produce general anesthesia and,
side effects. However, sexual dysfunction commonly occurs
therefore, are relatively safe drugs with a high therapeutic
with SSRIs (fluvoxamine, sertraline, and citalopram), as well as
index.
with TCAs, and SNRIs. Lithium is used for the treatment of
mania and bipolar disorder.

38. Which one of the following is a short-acting


hypnotic?
41. A 36-year-old man presents with symptoms of
a. Phenobarbital.
compulsive behavior. If anything is out of order, he
b. Diazepam.
c. Chlordiazepoxide. feels that “work will not be accomplished effectively or
efficiently.” He realizes that his behavior is interfering
d. Triazolam.
with his ability to accomplish his daily tasks but cannot
e. Flurazepam.
seem to stop himself. Which of the following drugs
would be most helpful to this patient?
a. Imipramine.
Answer: d. Triazolam. Triazolam is a short-acting drug. It has b. Fluvoxamine.
little daytime sedation. The other drugs listed are longer acting. c. Amitriptyline.
d. Tranylcypromine.
e. Lithium.
39. A 45-year-old man who has been injured in a car
accident is brought into the emergency room. His Answer: b. Fluvoxamine; SSRIs are particularly effective in
blood alcohol level on admission is 275 mg/dL. treating obsessive–compulsive disorder, and fluvoxamine is
Hospital records show a prior hospitalization for approved for this condition. The other drugs are less effective
alcohol-related seizures. His wife confirms that he in the treatment of obsessive–compulsive disorder.
has been drinking heavily for 3 weeks. What
treatment should be provided to the patient if he
goes into withdrawal? 42. Which antidepressant is the most sedating?
a. None. a. Fluoxetine.
b. Lorazepam. b. Duloxetine.
c. Pentobarbital. c. Nortriptyline.
d. Phenytoin. d. Citalopram.
e. Buspirone. e. Venlafaxine.

Answer: b. Lorazepam. It is important to treat the seizures Answer: c. Nortriptyline. Nortriptyline is the most sedating of
associated with alcohol withdrawal. Benzodiazepines, such as the list due to its histamine-blocking activity.
chlordiazepoxide, diazepam, or the shorter-acting lorazepam,
are effective in controlling this problem. They are less sedating
than pentobarbital or phenytoin. 43. An adolescent male is newly diagnosed with
schizophrenia. Which of the following antipsychotic
agents may have the best chance to improve his
apathy and blunted affect?
a. Chlorpromazine.
40. A 55-year-old teacher began to experience changes b. Fluphenazine.
in mood. He was losing interest in his work and
c. Haloperidol.
lacked the desire to play his daily tennis match. He
d. Risperidone.
was preoccupied with feelings of guilt, e. Thioridazine.
worthlessness, and hopelessness. In addition to the
Prepared by: MJCN
UIC PharmChem CARES
GREEN REVIEW 2021: Pharmacology I 51

generally begin in children aged 4 to 12 years. Diagnosis


includes obtaining an EEG that shows generalized 3-Hz waves.
Answer: d. Risperidone. Risperidone is the only antipsychotic
on the list that has some reported benefit in improving the
negative symptoms of schizophrenia. It is a second-generation
47. 52-year-old man has had several focal complex
antipsychotic, and the other drugs listed are firstgeneration
partial seizures over the last year. Which one of the
antipsychotic agents. All of the agents have the potential to
following therapies would be the most appropriate
diminish the hallucinations and delusional thought processes
initial therapy for this patient?
(positive symptoms).
a. Ethosuximide.
b. Levetiracetam.
c. Diazepam.
44. Which one of the following antipsychotics has been
d. Carbamazepine plus primidone.
shown to be a partial agonist at the dopamine D2 e. Watchful waiting.
receptor?
a. Aripiprazole. Answer: b. Levetiracetam. The patient has had many seizures,
b. Clozapine.
and the risks of not starting drug therapy would be
c. Haloperidol.
d. Risperidone. substantially greater than the risks of treating his seizures.
Because the patient has impaired consciousness during the
e. Thioridazine.
seizure, he is at risk for injury during an attack. Monotherapy
with primary agents is preferred for most patients. The
advantages of monotherapy include reduced frequency of
Answer: a. Aripiprazole.. Aripiprazole is the agent that acts as adverse effects, absence of interactions between antiepileptic
a partial agonist at D2 receptors. Theoretically, the drug would drugs, lower cost, and improved compliance. Ethosuximide
enhance action at these receptors when there is a low
and diazepam are not indicated for complex partial seizures.
concentration of dopamine and would block the actions of high
concentrations of dopamine. All of the other drugs are only
antagonistic at D2 receptors, with haloperidol being particularly
48. Which of the following is a potent analgesic but a
potent.
weak anesthetic?
a. Etomidate.
b. Halothane.
45. Which of the following antipsychotic agents is c. Midazolam.
available in a LAI formulation that may be useful for d. Nitrous oxide.
patients with difficulty adhering to therapy?
e. Thiopental.
a. Asenapine. f. Answer: Halothane.
b. Chlorpromazine.
c. Clozapine. Answer: d. Nitrous oxide. Etomidate is a hypnotic agent but
d. Quetiapine. lacks analgesic activity. Midazolam is a common
e. Risperidone. sedative/amnestic. Halothane and thiopental are potent
anesthetics with weak analgesic properties. Nitrous oxide
provides good analgesia but is a weak anesthetic that must be
Answer: e. Risperidone. Risperidone is available in a LAI combined with other agents to provide complete anesthesia.
formulation containing risperidone microspheres. The other
agents listed do not have LAI formulations. Aripiprazole,
fluphenazine, haloperidol, olanzapine, and paliperidone are 49. Which one of the following is a potent intravenous
other antipsychotics that are available in LAI formulations anesthetic but a weak analgesic?
a. Propofol.
b. Benzodiazepines.
46. A 9-year-old boy is sent for neurologic evaluation c. Ketamine.
because of episodes of apparent inattention. Over d. Fentanyl.
the past year, the child has experienced episodes e. Isoflurane.
during which he develops a blank look on his face Answer: a. Propofol. Propofol is a potent anesthetic but a
and his eyes blink for 15 seconds. He immediately weak analgesic. It is the most widely used intravenously
resumes his previous activity. Which one the administered general anesthetic. It has a high lipid solubility.
following best describes this patient’s seizures?
The other choices do not fit this profile.
a. Simple partial.
b. Complex partial.
c. Tonic–clonic.
d. Absence. 50. An 80-year-old patient with asthma and low blood
pressure requires anesthesia for an emergency
e. Myoclonic.
surgical procedure. Which of the following agents
Answer: d. Absence. The patient is experiencing episodes of would be most appropriate for inducing anesthesia in
absence seizures. Consciousness is impaired briefly and they this patient?
Prepared by: MJCN
UIC PharmChem CARES
GREEN REVIEW 2021: Pharmacology I 52

a. Desflurane.
b. Ketamine.
c. Propofol.
d. Thiopental.

Answer: b. Ketamine. Ketamine may be beneficial since it is a


potent bronchodilator and may not lower blood pressure like
other agents. Desflurane is an inhaled anesthetic that may
stimulate respiratory reflexes. It is used for maintenance, not
induction, and may lower blood pressure. Propofol may also
decrease blood pressure. Thiopental is a short-acting
barbiturate that can cause bronchospasm.

Prepared by: MJCN

You might also like